Sunteți pe pagina 1din 181

Preparation Booklet for MATH1051

.
.

How to use this book


You should identify those mathemati al skills in whi h you struggle with and spend extra time reading about and working through problems on those topi s. This book ontains material whi h is assumed knowledge in MATH1051. Gaps in your ba kground skills an seriously ae t your progress in
MATH1051.
Do the online diagnosti tests on the ourse Bla kboard site.
Identify the topi s you need help with.
Read the relevant hapter in the booklet.
Browse the problems at the end of the hapter and he k against the assignment worked solutions.

S hool of Mathemati s and Physi s, The University of Queensland, Brisbane QLD 4072, Australia

ii

Contents
List of Tables . . . . . . . . . . . . . . . . . . . . . . . . . . . . . . . . . . . . . . . . . . . .
List of Figures . . . . . . . . . . . . . . . . . . . . . . . . . . . . . . . . . . . . . . . . . . .
List of Symbols . . . . . . . . . . . . . . . . . . . . . . . . . . . . . . . . . . . . . . . . . . .
1

Notation and Sets

1.1
1.2
1.3
1.4
1.5
1.6
2

.
.
.
.
.
.

.
.
.
.
.
.

.
.
.
.
.
.

.
.
.
.
.
.

.
.
.
.
.
.

.
.
.
.
.
.

.
.
.
.
.
.

.
.
.
.
.
.

.
.
.
.
.
.

.
.
.
.
.
.

.
.
.
.
.
.

.
.
.
.
.
.

.
.
.
.
.
.

.
.
.
.
.
.

.
.
.
.
.
.

.
.
.
.
.
.

.
.
.
.
.
.

.
.
.
.
.
.

.
.
.
.
.
.

.
.
.
.
.
.

.
.
.
.
.
.

.
.
.
.
.
.

.
.
.
.
.
.

.
.
.
.
.
.

.
.
.
.
.
.

.
.
.
.
.
.

.
.
.
.
.
.

.
.
.
.
.
.

.
.
.
.
.
.

.
.
.
.
.
.

.
.
.
.
.
.

.
.
.
.
.
.

Introdu tion . . . . . . . . . .
Properties of sums . . . . . .
Pra ti e Problems . . . . . .
Answers to Pra ti e Problems
Introdu tion . . . . . . . . . .
Equivalent Fra tions . . . . .
Arithmeti of fra tions . . . .
Pra ti e Problems . . . . . .
Answers to Pra ti e Problems

1
1
4
5
6
7
9

.
.
.
.

.
.
.
.

.
.
.
.

.
.
.
.

.
.
.
.

.
.
.
.

.
.
.
.

.
.
.
.

.
.
.
.

.
.
.
.

.
.
.
.

.
.
.
.

.
.
.
.

.
.
.
.

.
.
.
.

.
.
.
.

.
.
.
.

.
.
.
.

.
.
.
.

.
.
.
.

.
.
.
.

.
.
.
.

.
.
.
.

.
.
.
.

.
.
.
.

.
.
.
.

.
.
.
.

.
.
.
.

.
.
.
.

.
.
.
.

.
.
.
.

.
.
.
.

Fra tions

3.1
3.2
3.3
3.4
3.5
4

Sigma Notation

2.1
2.2
2.3
2.4
3

Introdu tion . . . . . . . . . .
Set Notation . . . . . . . . . .
Interval Notation . . . . . . .
Set and Logi Notation . . . .
Pra ti e Problems . . . . . .
Answers to Pra ti e Problems

vii
viii
1

9
12
14
15
17

.
.
.
.
.

.
.
.
.
.

.
.
.
.
.

.
.
.
.
.

.
.
.
.
.

.
.
.
.
.

.
.
.
.
.

.
.
.
.
.

.
.
.
.
.

.
.
.
.
.

.
.
.
.
.

.
.
.
.
.

.
.
.
.
.

.
.
.
.
.

.
.
.
.
.

.
.
.
.
.

.
.
.
.
.

.
.
.
.
.

.
.
.
.
.

.
.
.
.
.

.
.
.
.
.

.
.
.
.
.

.
.
.
.
.

.
.
.
.
.

.
.
.
.
.

.
.
.
.
.

.
.
.
.
.

.
.
.
.
.

.
.
.
.
.

.
.
.
.
.

.
.
.
.
.

.
.
.
.
.

Fa torization

4.1 Introdu tion . . . . . . . . . . . . . .


4.2 Spe ial Cases . . . . . . . . . . . . .
4.2.1 Perfe t squares . . . . . . . .
4.2.2 The dieren e of two squares
4.3 Fa torising quadrati s . . . . . . . .
4.4 Pra ti e Problems . . . . . . . . . .
4.5 Answers to Pra ti e Problems . . . .

17
17
18
21
22
25

.
.
.
.
.
.
.

.
.
.
.
.
.
.

.
.
.
.
.
.
.
iii

.
.
.
.
.
.
.

.
.
.
.
.
.
.

.
.
.
.
.
.
.

.
.
.
.
.
.
.

.
.
.
.
.
.
.

.
.
.
.
.
.
.

.
.
.
.
.
.
.

.
.
.
.
.
.
.

.
.
.
.
.
.
.

.
.
.
.
.
.
.

.
.
.
.
.
.
.

.
.
.
.
.
.
.

.
.
.
.
.
.
.

.
.
.
.
.
.
.

.
.
.
.
.
.
.

.
.
.
.
.
.
.

.
.
.
.
.
.
.

.
.
.
.
.
.
.

.
.
.
.
.
.
.

.
.
.
.
.
.
.

.
.
.
.
.
.
.

.
.
.
.
.
.
.

.
.
.
.
.
.
.

.
.
.
.
.
.
.

.
.
.
.
.
.
.

25
28
28
28
29
31
32

Inequalities

5.1
5.2
5.3
5.4
5.5
5.6
6

.
.
.
.
.
.

.
.
.
.
.
.

.
.
.
.
.
.

.
.
.
.
.
.

.
.
.
.
.
.

.
.
.
.
.
.

.
.
.
.
.
.

.
.
.
.
.
.

.
.
.
.
.
.

.
.
.
.
.
.

.
.
.
.
.
.

.
.
.
.
.
.

.
.
.
.
.
.

.
.
.
.
.
.

.
.
.
.
.
.

.
.
.
.
.
.

.
.
.
.
.
.

.
.
.
.
.
.

.
.
.
.
.
.

.
.
.
.
.
.

.
.
.
.
.
.

.
.
.
.
.
.

.
.
.
.
.
.

.
.
.
.
.
.

.
.
.
.
.
.

.
.
.
.
.
.

.
.
.
.
.
.

.
.
.
.
.
.

.
.
.
.
.
.

.
.
.
.
.
.

Introdu tion . . . . . . . . . .
Index Laws . . . . . . . . . .
Pra ti e Problems . . . . . .
Answers to Pra ti e Problems

.
.
.
.

.
.
.
.

.
.
.
.

.
.
.
.

.
.
.
.

.
.
.
.

.
.
.
.

.
.
.
.

.
.
.
.

.
.
.
.

.
.
.
.

.
.
.
.

.
.
.
.

.
.
.
.

.
.
.
.

.
.
.
.

.
.
.
.

.
.
.
.

.
.
.
.

.
.
.
.

.
.
.
.

.
.
.
.

.
.
.
.

.
.
.
.

.
.
.
.

.
.
.
.

.
.
.
.

.
.
.
.

.
.
.
.

.
.
.
.

.
.
.
.

.
.
.
.

45
46
51
52
55

Introdu tion . . . . . . . . . .
The number e . . . . . . . . .
Logarithm Laws . . . . . . . .
Solving Log Equations . . . .
Pra ti e Problems . . . . . .
Answers to Pra ti e Problems

.
.
.
.
.
.

.
.
.
.
.
.

.
.
.
.
.
.

.
.
.
.
.
.

.
.
.
.
.
.

.
.
.
.
.
.

.
.
.
.
.
.

.
.
.
.
.
.

.
.
.
.
.
.

.
.
.
.
.
.

.
.
.
.
.
.

.
.
.
.
.
.

.
.
.
.
.
.

.
.
.
.
.
.

.
.
.
.
.
.

.
.
.
.
.
.

.
.
.
.
.
.

.
.
.
.
.
.

.
.
.
.
.
.

.
.
.
.
.
.

.
.
.
.
.
.

.
.
.
.
.
.

.
.
.
.
.
.

.
.
.
.
.
.

.
.
.
.
.
.

.
.
.
.
.
.

.
.
.
.
.
.

.
.
.
.
.
.

.
.
.
.
.
.

.
.
.
.
.
.

.
.
.
.
.
.

.
.
.
.
.
.

Introdu tion . . . . . . . . . . . . . . . . . . .
Radians . . . . . . . . . . . . . . . . . . . . .
Trigonometry of right-angled triangles . . . .
General denitions of trigonometri fun tions
Pra ti e Problems . . . . . . . . . . . . . . .
Answers to Pra ti e Problems . . . . . . . . .
Basi derivatives . . . . . . . .
Derivatives of sin(x) and cos(x)
Derivatives of ex and ln x . . .
Leibniz's Notation . . . . . . .
Anti-derivatives . . . . . . . . .
Pra ti e Problems . . . . . . .
Answers to Pra ti e Problems .

55
56
58
60
64
65
67

.
.
.
.
.
.

.
.
.
.
.
.

.
.
.
.
.
.

.
.
.
.
.
.

.
.
.
.
.
.

.
.
.
.
.
.

.
.
.
.
.
.

.
.
.
.
.
.

.
.
.
.
.
.

.
.
.
.
.
.

.
.
.
.
.
.

.
.
.
.
.
.

.
.
.
.
.
.

.
.
.
.
.
.

.
.
.
.
.
.

.
.
.
.
.
.

.
.
.
.
.
.

.
.
.
.
.
.

.
.
.
.
.
.

.
.
.
.
.
.

.
.
.
.
.
.

.
.
.
.
.
.

.
.
.
.
.
.

Derivatives and Anti-derivatives

9.1
9.2
9.3
9.4
9.5
9.6
9.7

35
35
38
39
42
42
45

Trigonometry

8.1
8.2
8.3
8.4
8.5
8.6
9

.
.
.
.
.
.

Logs

7.1
7.2
7.3
7.4
7.5
7.6
8

.
.
.
.
.
.

Powers

6.1
6.2
6.3
6.4
7

Introdu tion . . . . . . . . . .
Solving basi inequalities . . .
Combining inequalities . . . .
Absolute value . . . . . . . .
Pra ti e Problems . . . . . .
Answers to Pra ti e Problems

35

67
67
68
74
79
82
85

.
.
.
.
.
.
.

.
.
.
.
.
.
.

.
.
.
.
.
.
.

.
.
.
.
.
.
.

.
.
.
.
.
.
.

.
.
.
.
.
.
.

10 Ve tors

.
.
.
.
.
.
.

.
.
.
.
.
.
.

.
.
.
.
.
.
.

.
.
.
.
.
.
.

.
.
.
.
.
.
.

.
.
.
.
.
.
.

.
.
.
.
.
.
.

.
.
.
.
.
.
.

.
.
.
.
.
.
.

.
.
.
.
.
.
.

.
.
.
.
.
.
.

.
.
.
.
.
.
.

.
.
.
.
.
.
.

.
.
.
.
.
.
.

.
.
.
.
.
.
.

.
.
.
.
.
.
.

.
.
.
.
.
.
.

.
.
.
.
.
.
.

.
.
.
.
.
.
.

.
.
.
.
.
.
.

.
.
.
.
.
.
.

.
.
.
.
.
.
.

.
.
.
.
.
.
.

.
.
.
.
.
.
.

.
.
.
.
.
.
.

85
88
89
90
91
94
96
99

10.1 Position Ve tors . . . . . . . . . . . . . . . . . . . . . . . . . . . . . . . . . . . . . . . . 101


10.2 Denition (magnitude) . . . . . . . . . . . . . . . . . . . . . . . . . . . . . . . . . . . . 101
10.3 Ve tor Addition . . . . . . . . . . . . . . . . . . . . . . . . . . . . . . . . . . . . . . . . 102
iv

10.3.1 Properties . . . . . . .
10.4 S alar Multipli ation . . . . .
10.4.1 Properties . . . . . . .
10.5 Ve tor Subtra tion . . . . . .
10.6 Unit Ve tors . . . . . . . . . .
10.7 Ve tors in 3-spa e . . . . . . .
10.7.1 Notation . . . . . . . .
10.8 Dot Produ t . . . . . . . . . .
10.8.1 Denition . . . . . . .
10.8.2 Properties . . . . . . .
10.8.3 Component Form . . .
10.9 Pra ti e Problems . . . . . .
10.10Answers to Pra ti e Problems

.
.
.
.
.
.
.
.
.
.
.
.
.

.
.
.
.
.
.
.
.
.
.
.
.
.

.
.
.
.
.
.
.
.
.
.
.
.
.

.
.
.
.
.
.
.
.
.
.
.
.
.

.
.
.
.
.
.
.
.
.
.
.
.
.

.
.
.
.
.
.
.
.
.
.
.
.
.

.
.
.
.
.
.
.
.
.
.
.
.
.

.
.
.
.
.
.
.
.
.
.
.
.
.

.
.
.
.
.
.
.
.
.
.
.
.
.

.
.
.
.
.
.
.
.
.
.
.
.
.

.
.
.
.
.
.
.
.
.
.
.
.
.

.
.
.
.
.
.
.
.
.
.
.
.
.

.
.
.
.
.
.
.
.
.
.
.
.
.

.
.
.
.
.
.
.
.
.
.
.
.
.

.
.
.
.
.
.
.
.
.
.
.
.
.

.
.
.
.
.
.
.
.
.
.
.
.
.

.
.
.
.
.
.
.
.
.
.
.
.
.

.
.
.
.
.
.
.
.
.
.
.
.
.

.
.
.
.
.
.
.
.
.
.
.
.
.

.
.
.
.
.
.
.
.
.
.
.
.
.

.
.
.
.
.
.
.
.
.
.
.
.
.

.
.
.
.
.
.
.
.
.
.
.
.
.

.
.
.
.
.
.
.
.
.
.
.
.
.

.
.
.
.
.
.
.
.
.
.
.
.
.

.
.
.
.
.
.
.
.
.
.
.
.
.

.
.
.
.
.
.
.
.
.
.
.
.
.

.
.
.
.
.
.
.
.
.
.
.
.
.

.
.
.
.
.
.
.
.
.
.
.
.
.

.
.
.
.
.
.
.
.
.
.
.
.
.

.
.
.
.
.
.
.
.
.
.
.
.
.

.
.
.
.
.
.
.
.
.
.
.
.
.

.
.
.
.
.
.
.
.
.
.
.
.
.

11 Matri es

119

11.1 Introdu tion . . . . . . . . . . . . . . . .


11.2 Denition (matrix) . . . . . . . . . . . .
11.3 Equality . . . . . . . . . . . . . . . . . .
11.4 Addition . . . . . . . . . . . . . . . . . .
11.5 Properties . . . . . . . . . . . . . . . . .
11.6 S alar Multipli ation . . . . . . . . . . .
11.7 Matrix Multipli ation . . . . . . . . . .
11.8 Non- ommutativity of matri es . . . . .
11.9 Properties of Matrix Multipli ation . . .
11.10Transposition . . . . . . . . . . . . . . .
11.10.1 Properties of Transposition . . .
11.11Dot Produ t and Matrix Multipli ation .
11.12Square Matri es . . . . . . . . . . . . . .
11.13The Identity Matrix . . . . . . . . . . .
11.14Inverses . . . . . . . . . . . . . . . . . .
11.15Matrix Powers . . . . . . . . . . . . . .
11.16Pra ti e Problems . . . . . . . . . . . .
11.17Answers to Pra ti e Problems . . . . . .

.
.
.
.
.
.
.
.
.
.
.
.
.
.
.
.
.
.

.
.
.
.
.
.
.
.
.
.
.
.
.
.
.
.
.
.

.
.
.
.
.
.
.
.
.
.
.
.
.
.
.
.
.
.

.
.
.
.
.
.
.
.
.
.
.
.
.
.
.
.
.
.

.
.
.
.
.
.
.
.
.
.
.
.
.
.
.
.
.
.

.
.
.
.
.
.
.
.
.
.
.
.
.
.
.
.
.
.

.
.
.
.
.
.
.
.
.
.
.
.
.
.
.
.
.
.

.
.
.
.
.
.
.
.
.
.
.
.
.
.
.
.
.
.

.
.
.
.
.
.
.
.
.
.
.
.
.
.
.
.
.
.

.
.
.
.
.
.
.
.
.
.
.
.
.
.
.
.
.
.

.
.
.
.
.
.
.
.
.
.
.
.
.
.
.
.
.
.

.
.
.
.
.
.
.
.
.
.
.
.
.
.
.
.
.
.

.
.
.
.
.
.
.
.
.
.
.
.
.
.
.
.
.
.

.
.
.
.
.
.
.
.
.
.
.
.
.
.
.
.
.
.

.
.
.
.
.
.
.
.
.
.
.
.
.
.
.
.
.
.

.
.
.
.
.
.
.
.
.
.
.
.
.
.
.
.
.
.

.
.
.
.
.
.
.
.
.
.
.
.
.
.
.
.
.
.

.
.
.
.
.
.
.
.
.
.
.
.
.
.
.
.
.
.

.
.
.
.
.
.
.
.
.
.
.
.
.
.
.
.
.
.

.
.
.
.
.
.
.
.
.
.
.
.
.
.
.
.
.
.

.
.
.
.
.
.
.
.
.
.
.
.
.
.
.
.
.
.

.
.
.
.
.
.
.
.
.
.
.
.
.
.
.
.
.
.

.
.
.
.
.
.
.
.
.
.
.
.
.
.
.
.
.
.

.
.
.
.
.
.
.
.
.
.
.
.
.
.
.
.
.
.

.
.
.
.
.
.
.
.
.
.
.
.
.
.
.
.
.
.

.
.
.
.
.
.
.
.
.
.
.
.
.
.
.
.
.
.

12 Worked Solutions

12.1
12.2
12.3
12.4
12.5
12.6
12.7

Worked Solutions to Notation and Sets


Worked Solutions to Sigma Notation .
Worked Solutions to Fra tions . . . . .
Worked Solutions to Fa torisation . . .
Worked Solutions to Inequalities . . .
Worked Solutions to Powers . . . . . .
Worked Solutions to Logs . . . . . . .

103
103
104
104
105
106
108
110
110
111
112
114
116
119
119
121
121
122
124
125
128
130
131
132
132
133
135
136
137
138
140
143

.
.
.
.
.
.
.

.
.
.
.
.
.
.
v

.
.
.
.
.
.
.

.
.
.
.
.
.
.

.
.
.
.
.
.
.

.
.
.
.
.
.
.

.
.
.
.
.
.
.

.
.
.
.
.
.
.

.
.
.
.
.
.
.

.
.
.
.
.
.
.

.
.
.
.
.
.
.

.
.
.
.
.
.
.

.
.
.
.
.
.
.

.
.
.
.
.
.
.

.
.
.
.
.
.
.

.
.
.
.
.
.
.

.
.
.
.
.
.
.

.
.
.
.
.
.
.

.
.
.
.
.
.
.

.
.
.
.
.
.
.

.
.
.
.
.
.
.

.
.
.
.
.
.
.

.
.
.
.
.
.
.

.
.
.
.
.
.
.

.
.
.
.
.
.
.

.
.
.
.
.
.
.

.
.
.
.
.
.
.

143
144
145
149
153
156
157

12.8 Worked Solutions to Trigonometry . . . . . . . . . .


12.9 Worked Solutions to Derivatives and Anti-derivatives
12.10Worked Solutions to Ve tors . . . . . . . . . . . . . .
12.11Worked Solutions to Matri es . . . . . . . . . . . . .

vi

.
.
.
.

.
.
.
.

.
.
.
.

.
.
.
.

.
.
.
.

.
.
.
.

.
.
.
.

.
.
.
.

.
.
.
.

.
.
.
.

.
.
.
.

.
.
.
.

.
.
.
.

.
.
.
.

.
.
.
.

.
.
.
.

.
.
.
.

.
.
.
.

.
.
.
.

159
161
162
168

List of Tables
1.1 Spe ial Sets . . . . . . . . . . . . . . . . . . . . . . . . . . . . . . . . . . . . . . . . . .
1.2 Logi al symbols for sets . . . . . . . . . . . . . . . . . . . . . . . . . . . . . . . . . . .

2
6

5.1 The meaning of inequalities . . . . . . . . . . . . . . . . . . . . . . . . . . . . . . . . .

35

8.1 Angles and Ratios in Important Triangles . . . . . . . . . . . . . . . . . . . . . . . . .

70

vii

List of Figures
1.1 The real number line. . . . . . . . . . . . . . . . . . . . . . . . . . . . . . . . . . . . . .
10.1
10.2
10.3
10.4
10.5

Ve tor of unit length at 45 to x-axis has many representations as shown


Geometri representation of a ve tor . . . . . . . . . . . . . . . . . . . .


P Q + QR=P R . . . . . . . . . . . . . . . . . . . . . . . . . . . . . . . .
A ve tor in 3-spa e. . . . . . . . . . . . . . . . . . . . . . . . . . . . . . .
v gives the velo ity ve tor of the zeppelin. . . . . . . . . . . . . . . . . .

viii

.
.
.
.
.

.
.
.
.
.

.
.
.
.
.

.
.
.
.
.

.
.
.
.
.

.
.
.
.
.

.
.
.
.
.

.
.
.
.
.

4
100
100
102
107
110

Chapter 1
Notation and Sets

1.1 Introdu tion

There are several types of important notation with whi h you should
be familiar. Set notation allows a set to be des ribed without writing
out ea h of its elements. Interval notation does the same for ertain
sets of real numbers. Finally, sigma notation allows the sum of many
terms to be expressed in a on ise manner.
1.2 Set Notation

It is often useful to be able to des ribe a set, whi h is a olle tion of


elements. When the number of elements in a set is small, the set an
be des ribed by listing ea h of its elements. For instan e, we an dene
the sets A, B and C as follows:
A = {1, 2, 3},

B = {0, 4, 6},

C = {12, 7r, e}.

n o

Sets are always designated using the urly bra kets,


, and are
generally given a apital letter as a label.
There are many ases where the number of elements in a set makes it
impossible to des ribe the set by listing all its elements. Some of these
1

Chapter 1. Notation and Sets

sets have spe ial symbols. For instan e, the set of the ounting numbers
{1, 2, 3, . . . } is known as the natural numbers and is written as N.
Similarly, the set of all positive and negative natural numbers with
zero, {. . . , 3, 2, 1, 0, 1, 2, 3, . . . }, is the set of integers and is written as
Z. The set of all rational numbers, that is, those of the form a/b where
both a and b are integers, is denoted by Q. The set of all real numbers,
whi h in ludes all rational and irrational numbers, is denoted R (An
irrational number is one that annot be expressed in the from p/q, but
has a de imal expansion that is neither nite nor periodi ). The set of
omplex numbers is denoted C and onsists of all numbers of the form
a + bi , where a and b are real numbers, and i2 = 1.
One other important set is the empty set. It is dened as the set
with no elements: {} , and is denoted .
Table 1.1:

Symbol

N
Z
Q
R
C

Spe ial Sets

Meaning
the set {} with no elements
{1, 2, 3, . . . }

Name
Empty Set
Natural Numbers

{. . . , 3, 2, 1, 0, 1, 2, 3, . . . }

Integers

All numbers but omplex

{a + b 1 | a, b R}

Real Numbers

{ 11 , 12 , 13 , 23 , 23 , 2, . . . } Rational Numbers

Complex Numbers

A parti ular number will either belong to a given set or it will not.
For example, the number 4 belongs to the natural numbers, while the
number 2 does not. We use the notation 4 N and 2 6 N to denote

1.2 Set Notation

these statements. In general x A says that the element x is in the


set A, and x 6 A says that the element x is not in the set A.

Sets an be dened using a ondition of some form. For example,


suppose you wanted to des ribe the set of all integers that were greater
than 7. In this ase the notation
{x | x Z and x > 7}

is used. The line | an be read as `su h that', so that the set above is `the
set of all x su h that x is an integer and x is greater than 7. In general,
you an reate many sets of the form {x | x has ertain properties }.
The use of the letter x in this notation is merely onvention. This
notation also allows us to dene the omplex numbers more su in tly.
We now have C = {a + bi | a, b R}.
Example 1.2.1.

x 12}.

Solution:

List the elements in the set {x | x Z and 1

The integers from 1 to 12, that is


1, 0, 1, 2, 3, 4, 5, 6, 7, 8, 9, 10, 11, 12.

Example 1.2.2.

Des ribe the set of rational numbers in set nota-

tion.
Solution:
The rationals are all the numbers than an be represented as a fra tion, with an integer numerator and denominator.
So
na
o
Q=
| a, b Z, and b 6= 0 .
b

This next example demonstrates that the des ription of a set is not

Chapter 1. Notation and Sets

unique.
Example 1.2.3.

two other ways.


Solution:

Des ribe the set S = {2, 4, 6, 8} in set notation in

S = {x | x Z and x is even and 2 x 8},

= {x | x N and x is even and 1 x 9}.

1.3 Interval Notation

The real numbers an be represented as a line like the diagram below,


(Figure 1.1). In many ases it is ne essary to be able to dene intervals

Figure 1.1:

The real number line.

of this real number line. These onsist of all real numbers between 2
bounds. They have three types: losed intervals, open intervals and
half-open, half losed intervals.
A losed interval is an interval whi h in ludes its endpoints. So an
interval of the form {x | x R and a x b} is a losed interval
and we dene the notation [a, b] to des ribe this interval.
An open interval is an interval whi h does not in lude its endpoints.
For example, the set of all real numbers between 0 and 1 but not
in luding 0 or 1 is an open interval. Open intervals are of the form
{x | x R and a < x < b}, and we dene the notation (a, b) to

1.4 Set and Logi Notation

des ribe this interval.


Finally, a half-open, half- losed interval is an interval whi h in ludes
one of its endpoints. So any interval of the form {x | x R and a <
x b} or {x | x R and a x < b} is a half-open, half- losed
interval. We dene the notation (a, b] and [a, b) respe tively to des ribe
these intervals, where the square bra ket denotes the endpoint whi h
is in luded.
We also introdu e the onvenient symbol . The interval (a, )
onsists of all real numbers between a and . The interval (, ) =
R, For the set of all the real numbers greater than or equal to a number,
a, you ould use a half-open, half- losed interval. In that ase {x | x
R and a x} would be denoted [a, ).
1.4 Set and Logi Notation

In your mathemati s ourses you may o asionally en ounter the following logi al symbols asso iated with dis ussing sets. The symbols
and are usually reserved for use with logi , set theory, and topology, and we mean `for all' and `there exists' respe tively. However, in
mathemati s le tures you might see these symbols as shorthand.

Chapter 1. Notation and Sets

Table 1.2:

Symbol

Logi al symbols for sets

Meaning
Example

belongs to
2 {1, 2, 3}
6
does not belong to
8 6 {1, 2, 3}

for all
x R, x2 0

there exists
x R | x > 4
6
there does not exist
6 x R | x2 < 0
=
implies (if, then)
x Z and x > 0 = x N
or i
if and only if
x Z and x > 0 i x N

is a subset of
NZ

is a subset of or is equal to
{1, 2, 3} {1, 2, 3}

the union of
{1, 0} N = {1, 0, 1, 2, 3, . . . }

the interse tion of


{1, 0, 1, 2, 3} {x | x Z and x > 1} = {2, 3}

set of pairs in ea h
{1, 2} {7, 2} = {(1, 7), (1, 2), (2, 7), (2, 2)}

1.5 Pra ti e Problems

Here are some problems for you to pra ti e, followed by answers.


Q 1.5.1.
Q 1.5.2.

Does 0 ?

List the elements of the following sets.

1. {x | x Z and 2 x 6}

2. {x | x Z and 3 x 0}
3. {x | x N and x < 0}

Q 1.5.3.

Use set notation with onditions to des ribe the following

sets.
1. {3, 4, 5, 6, 7, 8}

2. {4, 4, 0, 2}

3. {6, 5, 3, 4, 5}
4. {4, 5, 6, . . . }

1.6 Answers to Pra ti e Problems

Q 1.5.4.

Use interval notation to des ribe the following.

1. All the real numbers between 3 and 5 in luding 3 and 5.


2. {x | x R and 7 x < 15}
3. {x | x R and x > 4}
4. {x | x R and x 0}
1.6 Answers to Pra ti e Problems
A 1.6.1.

(Q 1.5.1) No. is the empty set.

A 1.6.2.

(Q 1.5.2)

1. {2, 3, 4, 5, 6}.
2. {2, 1, 0}.
3. This set has no elements (therefore {x | x N and x < 0} = ).
(Q 1.5.3) There are generally many ways to des ribe a set
in this fashion. Below is one example for ea h problem.
A 1.6.3.

1. {x | x Z and 3 x 8}.
2. {x | x Z and x is even and 4 x 2}.
3. {x | x Z and 6 x 5 or 3 x 5}.
4. {x | x N and 3 < x}.
A 1.6.4.

1. [3, 5].

(Q 1.5.4)

Chapter 1. Notation and Sets

2. [7, 15).
3. (4, ).

4. (, 0].

Chapter 2
Sigma Notation

2.1 Introdu tion

It is easy to write out expressions involving the sum of a few terms.


For example, 3x + 4x + 5x and 4 + 12x + x2 an easily be written by
hand. There are, however, many ases where we want to write down
the sum of many terms and it be omes far too long to write in this
fashion. One way to deal with this is to use a series of dots (an ellipsis)
to represent terms that are left out, as in 1 + 2 + + 64. The problem
with this notation is that it is often un lear what should be in luded
in the spa e left by the dots. Was that the sum, for instan e, of all
the numbers between 1 and 64, or all the powers of 2 from 20 to 26?
Both of these interpretations ould make sense on this o asion. To
over ome these problems, sigma notation is used, also referred to as
P
summation notation. The letter is a Greek apital sigma standing
for sum. The notation
64
X
i=1

represents the sum of all the numbers between 1 and 64, while
6
X
n=0

2n

10

Chapter 2. Sigma Notation

represents the sum of all powers of 2 from 0 to 6. In general, sigma


notation is of the form
upper
X

expression

lower

In this notation, lower usually has the form of variable = integer and
upper is usually a larger integer. Together lower and upper dene an
interval of integers that the variable will take on. The expression is
usually some formula involving the variable. The expression is evaluated for ea h of the possible integer values of the variable advan ing by
1 ea h time and the resulting terms are added together. For example,
5
X
j=2

(2j + 3) = (2 2 + 3) + (2 3 + 3) + (2 4 + 3) + (2 5 + 3),
= 7 + 9 + 11 + 13,
= 40.

The variable used in the notation is usually referred to as a dummy


variable, as hanging its name has no ee t on the sum the notation
represents.
Example 2.1.1.
P4
n=1 (n/5).

Solution:

Write out the sums des ribed by

P4

m=1 (m/5)

and

Both expressions represent the same sum,


1 2 3 4
+ + + = 2.
5 5 5 5

The next example is used to demonstrate that there is always more


than one way in whi h a sum an be written in sigma notation. How-

11

2.1 Introdu tion

ever, a sum in sigma notation will only orrespond to one expanded


sum.

Write 1 + 3 + 7 + 9 using sigma notation.


Solution:
In sigma notation, the value of the variable always
in reases by 1 at ea h step. Here we need the values of the terms
in the sum to in rease by 2 ea h time. If the variable takes on the
values 1, 2, 3 and so on in turn, then we an make the terms in the
sum in rease by 2 ea h time by multiplying the variable by 2. So we
an think of the terms as
Example 2.1.2.

2 1 1, 2 2 1, 2 3 1, . . . , 2 5 1,

so the sum is

5
X
(2n 1),
n=1

or we an think of the terms as

2 0 + 1, 2 1 + 1, 2 2 + 1, . . . , 2 4 + 1,

so the sum is

5
X

(2m + 1).

m=0

In fa t it is often onvenient to be able to view a sum in two ways.


For example, if n = m + 1, then we have
5
X
n=1

(2n 1) =

5
X

m+1=1

(2(m + 1) 1) =

5
X

m=0

(2m + 2 1) =

5
X

(2m + 1).

m=0

In the next example, are must be taken to determine whi h variable

12

Chapter 2. Sigma Notation

is taking the values from 1 to 5.

What is the expanded form of the sum 5i=1 ij ?


This orresponds to j + 2j + 3j + 4j + 5j .
P

Example 2.1.3.

Solution:

The variable an also be used in subs ripts.

What is the expanded form of the sum


This expands to i1 + i2 + i3 + i4 + i5.

Example 2.1.4.

Solution:

Example 2.1.5.

Evaluate N =

Solution:
N =

4
X

P4

n=1 (1)

P5

j=1 ij

n.

n 2

(1)nn2,

n=1

= (1)112 + (1)222 + (1)332 + (1)442,


= 1 + 4 9 + 16,
= 10.

2.2 Properties of sums

There may be a few rules whi h might fa ilitate adding in sigma notation. These are:
Rule 1.
n
X
j=1

kj = k

n
X

j=1

If we are summing over a variable j , another letter or number multiplied by j in the expression may be fa torised to the front of the sum.
In more familiar terms, k + 2k + 3k + + kn = k(1 + 2 + 3 + + n),

13

2.2 Properties of sums

or multipli ation distributes over addition.

Rule 2.

Let f (j) be a fun tion and a, n Z. Then


n
X
j=1

f (j) =

n+a1
X
j=a

f (j a + 1)

That is we an adjust the starting and ending points of the sum ( arefully).

Rule 3.

Let f (i) and g(j) be fun tions and m, n Z. Then


n
X
i=1

f (i) + g(j) =

n
X

f (i) +

i=1

n
X

g(j)

i=1

This is the asso iative law of addition: (a + b) + c = a + (b + c).

Rule 4.

Let Let m R be a onstant. Then


n
X

m = mn

i=1

Example 2.2.1.

Evaluate the sum

P5

j=1 (j

2j + 7).

14

Chapter 2. Sigma Notation

Solution:
5
5
5
5
X
X
X
X
3
3
(j 2j + 7) =
j +
(2j) +
7,
j=1

j=1
5
X
j=1
3

j=1
5
X

j3 2

j=1
3

j=1

(j) + 7 5,

= (1 + 23 + 3 + 43 + 53)
2 (1 + 2 + 3 + 4 + 5) + 35,

= (1 + 8 + 27 + 64 + 125) 2 15 + 35,
= 225 30 + 35,
= 220.

2.3 Pra ti e Problems

Here are some problems for you to pra ti e on, followed by answers.
Q 2.3.1.

1.
2.
3.

Write the following sums in expanded form.

P3

n=1 2n

P2

n=1 3

P5

i=0 i(i

Q 2.3.2.

+ 2)

Write the following sums in sigma notation.

1. 3 + 4 + 5 + 6 + 7 + 8
2. 2 + 1 + 4 + 7
1
3. 1 + 12 + 13 + . . . 100

15

2.4 Answers to Pra ti e Problems

Q 2.3.3.

Write the following sums in expanded form.


6
X
(1)j (2j + nj2 ).
j=3

Q 2.3.4.

Write 3x1 5x5 + 7x3 9x4 in sigma notation.

2.4 Answers to Pra ti e Problems


A 2.4.1.

To Q 2.3.1.

1. 2 + 4 + 6 = 12.
2. 13 + 1 + 3 + 9 = 403 .
3. 0 + 3 + 8 + 15 + 24 + 35.
To Q 2.3.2. There are many possible answers to these
problems. Those presented below are just examples.
A 2.4.2.

1.
2.
3.

P8

P100

i=3 i

P3

i=0 (3i
1
i=1 i

A 2.4.3.
A 2.4.4.

2).

To Q 2.3.3. (6 + n) + (8 + n2) (10 + n3) + (12 + n4).


To Q 2.3.4.

P4

i=1 (1)

i1

(2i + 1)xi.

Chapter 3
Fra tions

3.1 Introdu tion

A fra tion ontains two parts, a numerator and a denominator. In


the fra tion ab , the numerator is a and the denominator is b. This
represents the quantity a b. It is not possible to divide by zero, so
the denominator of a fra tion is not allowed to be zero.
3.2 Equivalent Fra tions

The fra tions 13 and 26 are equivalent, that is, both fra tions represent
the same amount. If you take any fra tion and multiply or divide both
the numerator and the denominator by the same (non-zero) quantity,
you obtain an equivalent fra tion. For example,
2
2 24
8
8
=
sin e =
= .
3 12
3 3 4 12

Equivalent fra tions help you to write a fra tion in its simplest form,
also known as lowest terms. To nd the simplest form of a fra tion
you must divide out any ommon fa tors from the numerator and denominator. For example, 14
= 147
= 32 . There are no ommon fa tors
21
217
between 2 and 3, so 23 is the simplest form of the fra tion. Unless told
otherwise, you should always express fra tions in their simplest form.
17

18

Chapter 3. Fra tions

When you write a fra tion that involves variables (an algebrai fra tion) in simplest terms, be areful to only divide out ommon fa tors
and not single terms. For example,
4 + 2x 2(2 + x) 2 + x
=
=
.
6
2(3)
3

Some students are tempted to divide only part of the numerator by 2


and write 4+2x
= 2+2x
. This is WRONG.
6
3
Example 3.2.1.

Solution:

40xy
64x

(provided that x 6= 0).

40xy 8x 5y
= .
64x 8x
8

Example 3.2.2.

Solution:

Simplify

Simplify

9a+27
6a+18

(provided that a 6= 3).

9a + 27 9(a + 3) 9(a + 3) 3(a + 3) 3


=
=
= .
6a + 18 6(a + 3) 6(a + 3) 3(a + 3) 2

One point that must always be remembered is that we annot divide


by 0. Thus in Example 3.2.1 the answer is only valid provided that
x 6= 0. Again, in Example 3.2.2 the answer is only valid if a 6= 3.
3.3 Arithmeti of fra tions

Fra tions an be added, subtra ted, multiplied and divided. Although


many al ulators an perform al ulations with fra tions, it is important that you know how to manipulate fra tions, be ause when a fra tion involves a variable you annot simply 'put it in your al ulator'.
To add or subtra t fra tions they must have the same denomina-

19

3.3 Arithmeti of fra tions

tor. To determine the sum of two fra tions write ea h fra tion as an
equivalent fra tions so that the two fra tions have the same denominator, then add the numerators together and pla e that sum over the
ommon denominator. Finally, if ne essary, redu e your answer to its
simplest form.

Simplify 16 + 83 .
Solution:
To nd a ommon denominator, we need a number
that is divisible by both 6 and 8. Here we use 24.
Example 3.3.1.

1 3
4
9
13
+ =
+
= .
6 8 24 24 24

Note that we ould have used a dierent ommon denominator, for


example 48.
Simplify 5a
+ 4a
(provided that c, d 6= 0).
6d
3c
Solution:
The ommon denominator must be divisible by 3c
and by 6d, so we use 6cd.
Example 3.3.2.

5a 4a 5ac 8ad 5ac + 8ad


+
=
+
=
.
6d 3c 6cd 6cd
6cd

Subtra tion is very similar to addition ex ept that the numerators


are subtra ted, instead of added. Remember that for subtra tion you
still require a ommon denominator.
Example 3.3.3.

Solution:
Example 3.3.4.

Simplify 56 12 .
5 1 5 3 53 2 1
= =
= = .
6 2 6 6
6
6 3

Simplify

4x
7y

3
14x
(provided that x, y 6= 0).

20

Chapter 3. Fra tions

Solution:
4x
3
4x 2x
3y
8x2
3y
8x2 3y

=
.
7y 14x 7y 2x 14x y 14xy 14xy
14xy

To multiply two fra tions, multiply the numerators to obtain the


new numerator and multiply the denominators to obtain the new denominator. (There is no need for the denominators to be the same.)
Finally, redu e your answer to simplest form if ne essary.
Example 3.3.5.

Solution:
Example

Solution:

Simplify

5
12

45 .

5
4
54
4
1
=
=
= .
12 5 12 5 12 3
ac
2
3.3.6. Simplify 6 ba (provided that a, b 6= 0).

ac
2
2ac
c

=
= .
6
ba 6ba 3b
Dividing by a number d is the same as multiplying by the number d1 .

The re ipro al of a fra tion is obtained by inverting it, that is, swapping its numerator and denominator. To divide two fra tions, multiply
the rst fra tion by the re ipro al of the se ond fra tion. Remember
to redu e your answer to simplest form if ne essary.
Example 3.3.7.

Solution:
Example

Solution:

Simplify 47 118 .

4
8
4 11 4 11 11

=
=
= .
7 11 7
8
78
14
5ab
15b
3.3.8. Simplify 7c 6a (provided that a, b, c 6= 0).

5ab 15b 5ab


6a
5ab (2 3)a 2a2

=
=
.
7c
6a
7c
15b
7c (3 5)b
7c

3.4 Pra ti e Problems

21

3.4 Pra ti e Problems

Here are some problems for you to pra ti e on, followed by answers.
Q 3.4.1.

1.

14x
18x

2.

9ab
27bc

3.

8+4e
2e

(x 6= 0)

(b, c 6= 0)
(e 6= 0)

Q 3.4.2.

1.

3
12

Write ea h of the following fra tions in simplest form.

Write ea h of the following fra tions in simplest form.

+ 14

2. 13 + 154
3. 67 23
4.

13
18

79

Q 3.4.3.

Cal ulate the following and write your answer in simplest

form.
1.

4
11

2.

7
12

3
13

89

3. 58 27
4. 38 56

Q 3.4.4.

Cal ulate the following and write your answer in simplest

form.
1.

ab
4

2c
+ 8d
(d 6= 0)

22

Chapter 3. Fra tions

2.

4xy
9

3.

30gh
14

4.

9a
24

+ 3x
12
5h
(g 6= 0)
7g

8b5 (b 6= 0)

Q 3.4.5.

Cal ulate the following and write your answer in simplest

form.
1.

12g
16h

2.

4e
45

9d
(f 6= 0)
f

3.

xy
7

x
14
(x 6= 0)

4.

5a
7

7
9a
(a 6= 0)

h2 (h 6= 0)

3.5 Answers to Pra ti e Problems


A 3.5.1.

1.

7
9

2.

a
3c

3.

4+2c
e

A 3.5.2.

1.

1
2

2.

3
5

3.

4
21

4.

1
18

(Q 3.4.1)

(Q 3.4.2)

3.5 Answers to Pra ti e Problems

A 3.5.3.

1.

7
33

2.

27
104

3.

5
28

4.

9
20

A 3.5.4.

(Q 3.4.3)

(Q 3.4.4)

1.

abd+c
4d

2.

16xy+9x
36

3.

15g 2h5h
7g

4.

3ab5
8b

A 3.5.5.

1.

3g
8

2.

4ed
5f

3. 2y
4.

45a2
49

(Q 3.4.5)

23

Chapter 4
Fa torization

4.1 Introdu tion

Suppose you were asked to evaluate 2(3 + 4), using the orre t order
of operations (sometimes alled BEDMAS). You would rst add the
numbers in the bra kets (giving 7) and then multiply 2 by 7. Your
steps would be
2(3 + 4) = 2(7) = 14.
However we an evaluate this expression in a dierent way. We rst
multiply the number outside the bra kets by ea h term inside the bra kets, and then add the results. This is alled expanding, and the steps
are
2(3 + 4) = 2 3 + 2 4 = 6 + 8 = 14.
So given a produ t of terms involving bra kets, like a(b + c), it is easy
to multiply them together to expand the produ t, giving ab + ac.
Example 4.1.1.

Solution:

Expand 3(7 2).

3(7 2) = 3 7 3 2 = 21 6 = 15.
Example 4.1.2.

Expand 2x(4 + y).


25

26

Chapter 4. Fa torization

Solution:
2x(4 + y) = 2x 4 + 2x y = 8x + 2xy.

The opposite pro ess to expanding is alled fa torising. Given a sum


of terms, we try to nd a way to rewrite the sum as a produ t involving
bra kets. To do this we look for a ommon fa tor in the terms, whi h
is an expression that evenly divides into all or some of the terms. Then
the ommon fa tor goes outside the bra kets. For example, onsider
the expression 32 + 24. These two numbers are both divisible by 8,
whi h is their highest ommon fa tor. We an now write 32 + 24 as
produ ts of 8, so
32 + 24 = 8 4 + 8 3 = 8(4 + 3).

The fa tored form of 32 + 24 is 8(4 + 3). Note that you an always


he k your answer by expanding.
Example 4.1.3.

Fa torise 3x2 + 6x.

Solution:
3x2 + 6x = 3x x + 3x 2 = 3x(x + 2).
Example 4.1.4.

Fa torise 2ab + 8b + 14bc.

Solution:
2ab + 8b + 14bc = 2b a + 2b 4 + 2b 7c = 2b(a + 4 7c).
Example 4.1.5.

Solution:

Fa torise 13n + 9d.


There are no ommon fa tors in these two terms,

27

4.1 Introdu tion

so the expression annot be fa torised.


You an also fa torise an expression that in ludes terms that have
already been arranged in fa tored form. For example, 2(x+1)+y(x+1)
has two terms whi h have the ommon fa tor of x + 1. This expression
an be fa torised further giving the expression (x + 1)(2 + y). This
pro ess will be ome useful in fa toring quadrati s.

Example 4.1.6.

Fa torise 3xy + 6y + 2x + 4.

Solution:
3xy + 6y + 2x + 4 = 3y(x + 2) + 2(x + 2) = (3y + 2)(x + 2).

To expand expressions like (x + 2)(3y + 2) you must multiply ea h


term in the rst bra kets by ea h term in the se ond bra kets. This
method is sometimes alled FOIL, be ause you multiply the rst terms
(F), the outer terms (0), the inner terms (I) and the last terms (L).
For example,
(x + 2)(3y + 2) = x 3y + x 2 + 2 3y + 2 2,
= 3xy + 2x + 6y + 4.

Example 4.1.7.

Solution:

Expand (x 3)(y + 4).

(x3)(y +4) = xy +x4+(3)y +(3)4 = xy +4x3y 12.

28

Chapter 4. Fa torization

4.2 Spe ial Cases

There are some spe ial fa torisations that are useful to remember.
They are perfe t squares and the dieren e of two squares.
4.2.1 Perfe t squares

Here are two perfe t squares in expanded and fa torised form.


a2 + 2ab + b2 = (a + b)(a + b) = (a + b)2.

For example x2 + 4x + 4 = (x + 2)2 by putting x = a, b = 2.

a2 2ab + b2 = (a b)(a b) = (a b)2.

For example x2 6x + 9 = (x 3)2 by putting x = a, b = 3.

Example 4.2.1.

Fa torise a2 + 8a + 16.

Solution:
a2 + 8a + 16 = (a + 4)(a + 4) = (a + 4)2.

It helps to re ognise that the rst and last terms themselves are perfe t squares, then to onsider the middle term.
Example 4.2.2.

Solution:

Fa torise x2 4xy + 4y2 .

x2 4xy + 4y 2 = (x 2y)(x 2y) = (x 2y)2.


4.2.2 The dieren e of two squares

a2 b2 = (a + b)(a b).

For example, x2 49 = (x + 7)(x 7) by putting x = a, b = 7.

29

4.3 Fa torising quadrati s

Example 4.2.3.

Solution:

Fa torise x2 1.

x2 1 = (x)2 (1)2 = (x + 1)(x 1).


Example 4.2.4.

Solution:

Fa torise 4x2 25.

4x2 25 = (2x)2 (5)2 = (2x + 5)(2x 5).

4.3 Fa torising quadrati s

A quadrati is an expression that an be written in the form


ax2 + bx + c,

where a, b, and c are onstants (with a 6= 0) and x is the unknown


variable. We say a is the oe ient of x2, b is the oe ient of x and c
is the onstant term. In a quadrati , b and/or c an be zero, however
a an never be zero, otherwise we would have bx + c, whi h is a linear
expression.
If a quadrati fa torises, it must break up into two linear fa tors.
For example,
x2 + 8x + 7 = (x + 7)(x + 1).
Let's see how to fa torise this quadrati . We are trying to rewrite
x2 + 8x + 7 in the form (x + d)(x + e), where d and e are unknown
numbers. Now (x + d)(x + e) expands to x2 + (d + e)x + de. Thus we
have x2 +8x+7 = x2 +(d+e)x+de. Equating oe ients, we need to
nd d and e su h that d + e = 8 and de = 7. Sin e the only fa tors of

30

Chapter 4. Fa torization

7 are 1 and 7, we should take d = 7 and e = 1 (or vi e versa). Now we

an write the quadrati in fa tored form, x2 + 8x + 7 = (x + 7)(x + 1).


Note that (d + e) is the x oe ient b and de is the onstant term c
in the general quadrati expression.

Fa torise x2 + 8x + 12.
Solution:
First we need to nd two numbers that multiply to 12
and add to 8. There are many fa tors of 12: (12, 1), (6, 2) and (4, 3),
but only one of these ombinations adds to 8. That ombination is 2
and 6. Therefore,
Example 4.3.1.

x2 + 8x + 12 = x2 + 2x + 6x + 12,
= x(x + 2) + 6(x + 2),
= (x + 6)(x + 2).

Fa torise x2 + x 20.
Solution:
First we need to nd two numbers that multiply to
20 and add to 1. These are 4 and 5. Therefore,
Example 4.3.2.

x2 + x 20 = x2 + 5x 4x 5 (4),
= x(x + 5) 4(x + 5),

= (x 4)(x + 5).

When the oe ient of x2 is not 1, the method only hanges slightly.
To fa torise ax2 + bx + c, we still need to nd two numbers that add
to give b, but now they must multiply to give a c. For example,
to fa torise 2x2 + 9x + 4, two numbers that multiply to give 8 and
add to give 9 are 8 and 1. Written in expanded form the expression

31

4.4 Pra ti e Problems

2x2 + 9x + 4 be omes 2x2 + 8x + x + 4, whi h this is easily fa torised

into x(2x + 1) + 4(2x + 1) and then (2x + 1)(x + 4).

Fa torise 10x2 + 11x + 3.


Solution:
First we need to nd two numbers that multiply to
give 10 3 = 30 and add to give 11. These two numbers are 5 and
6. Therefore,
Example 4.3.3.

10x2 + 11x + 3 = 10x2 + 5x + 6x + 3,


= 5x(2x + 1) + 3(2x + 1),
= (5x + 3)(2x + 1).

4.4 Pra ti e Problems

Here are some problems for you to pra ti e on, followed by answers.
Q 4.4.1.

Fa torise the following.

1. 4y2 16y
2. 32ab + 16b + 8abc
3. 6xyz + 3yz + 18wyz
Q 4.4.2.

Fa torise the following.

1. 4 + 4b + b2
2. 25a2 10ad + d2
3. x2 + 12xy + 36y2
4. 144 48a + 4a2

32
Q 4.4.3.

Chapter 4. Fa torization

Fa torise the following.

1. 9y2 36
2. 64 121g4
3. 16x6 49y8
Q 4.4.4.

Fa torise the following.

1. x2 5x + 4
2. x2 + 2x 15
3. y2 3y + 2
4. x2 + 13x + 42
5. x2 11x + 24
6. a2 + 7a + 12
Q 4.4.5.

Fa torise the following.

1. 3x2 + 17x + 10
2. 2y2 + 12y + 16
3. 4x2 8x 12
4. 2z 2 z 15
4.5 Answers to Pra ti e Problems
A 4.5.1.

(Q 4.4.1)

1. 4y(y 4)

4.5 Answers to Pra ti e Problems

2. 8b(4a + 2 + ac)
3. 3yz(2x + 1 + 6w)
A 4.5.2.

(Q 4.4.2)

1. (2 + b)2
2. (5a d)2
3. (x + 6y)2

4. (12 2a)2
A 4.5.3.

(Q 4.4.5)

1. (3y + 6)(3y 6)

2. (8 + 11g2)(8 11g2)

3. (4x3 + 7y4)(4x3 7y4)


A 4.5.4.

(Q 4.4.5)

1. (x 4)(x 1)
2. (x 3)(x + 5)
3. (y 2)(y 1)
4. (x + 7)(x + 6)

5. (x 8)(x 3)
6. (a + 3)(a + 4)
A 4.5.5.

(Q 4.4.5)

1. (x + 5)(3x + 2)

33

34

2. (y + 4)(2y + 4)
3. 4(x 3)(x + 1)
4. (z 3)(2z + 5)

Chapter 4. Fa torization

Chapter 5
Inequalities

5.1 Introdu tion

Here are three examples of inequalities.


5 > 2,

x > 1,

17 < 23.

Inequalities arise when you are omparing two values or expressions


where one is 'greater than' or 'less than' the other. The symbols used
in these expressions are outlined below.
Table 5.1:

The meaning of inequalities

Symbol
Meaning
Examples
<
is less than
2<3

is less than or equal to 3 3 and 2 3


>
is greater than
3>2

is greater than or equal to 3 3 and 3 2


5.2 Solving basi inequalities

When asked to solve an inequality you need to determine the set of


values for the variable that make the inequality true. Inequalities are
solved in a similar way to equations (so whatever you do to one side
you must also do to the other) with three additional rules:
35

36

Chapter 5. Inequalities

1. If you multiply or divide both sides of the inequality by the same


negative number, then the inequality sign is reversed. (For example, if a > b then a < b.)
2. If you swap the quantities on the left and right sides then the
inequality sign is reversed. (For example, if a < b then b > a.)
3. If you invert the quantities on the left and right sides then the
inequality sign is reversed. (For example, if 2 < 3 then 12 > 13 .
For example, when both sides of x > 5 are multiplied by 1, the
result is x < 5. Note that if you multiply or divide by a variable you
must onsider two ases: if the variable is negative and if the variable
is positive.
Example 5.2.1.

Solution:

Solve 2x 6 < x + 3.
now add 6 to both sides
now subtra t x from both sides

2x 6 <x + 3
2x <x + 9
x <9

The solution to 2x 6 < x + 3 is x < 9. So any value of x satisfying


x < 9 makes the inequality 2x 6 < x + 3 true. For example, if
x = 7, then 14 6 < 7 + 3 is true.
Example 5.2.2.

Solve

58x
3

15.

37

5.2 Solving basi inequalities

Solution:
5 8x
15
3
5 8x 45

now multiply both sides by 3

8x 40

x 5.

The solution to

58x
3

now subtra t 5 from both sides


now divide both sides by 8 & reverse
15 is x 5.

Solve 4+a
< 5.
a
Solution: Note that here a annot be zero sin e we annot divide by
zero. If a > 0 then multiplying both sides by a gives
Example 5.2.3.

4 + a < 5a,
4 < 4a,
1 < a.

For this ase we want the values of a that satisfy both a > 0 (our
assumption) and a > 1 (the solution); these ombine to give a > 1.
If a < 0 then multiplying both sides by a gives
4 + a > 5a, reversed sin e multiplying by a negative
4 > 4a,
1 > a.

For this ase we want the values of a that satisfy both a < 0 (our
assumption) and a < 1 (the solution); these ombine to give a < 0.
The solution to 4+a
a < 5 is the real numbers a su h that a > 1 or

38

Chapter 5. Inequalities

a < 0.

5.3 Combining inequalities

The expression 5 < x < 14 is a ombined inequality. It states


that x is greater than 5 and also less than or equal to 14. These
inequalities ould be written separately as x > 5 and x < 14. When
two inequalities des ribe only one se tion on the number line, then the
inequalities an be ombined into a single statement. On the gure
below the solid dot shows that 14 is in luded (as x is less than or
equal to 14), while the open dot shows that 5 is ex luded (as x is
greater than 5).

However, if two inequalities des ribe two se tions of the number line,
then they annot be ombined into a single statement. An example of
two inequalities that annot be ombined is x > 2 or x < 3. These
inequalities reate two dierent se tions on the number line: x greater
than or equal to 2, and x less than 3. We annot write 2 < x < 3
be ause if 2 < x and x < 3 then 2 < 3, whi h is wrong.

Example 5.3.1.

bined?
Solution:

Can the inequalities x < 2 and x > 5 be om-

Yes, 5 < x < 2.

39

5.4 Absolute value

Example 5.3.2.

bined?
Solution:

Can the inequalities a > 67 or a > 1 be om-

No.

Can the inequalities a > 7 or a < 1 be ombined?


5.4 Absolute value

The notation | x | denotes the absolute value of the number x. This


means the distan e between 0 and x on the real number line. Thus
| 4 |= 4 and | 3 |= 3. The formal denition of absolute value is:
(
x if x 0,
| x |=
x if x < 0
Thus the absolute value strips away the negative sign (if present).
When solving inequalities with absolute value signs you must onsider
two ases: the ase when the expression inside the absolute value is
greater than or equal to zero, and the ase when it is less than zero.
Example 5.4.1.

Solve | x |< 4.

Solution:
Case 1: If x 0, then | x |= x so | x |< 4 means x < 4.
For this ase we want the values of x that satisfy both x 0 (our
assumption) and x < 4 (our solution); these give 0 x < 4.
Case 2: If x < 0, then | x |= x so | x |< 4 means x < 4,

40

Chapter 5. Inequalities

that is, x > 4. For this ase we want the values of x that satisfy
both x < 0 (our assumption) and x > 4 (our solution); these give
4 < x < 0.
The solution to | x |< 4 is 0 x < 4 or 4 < x < 0. We an put
these together as 4 < x < 4. The solution set is all values of x at
a distan e of less than 4 from the origin.

Example 5.4.2.

Solve | 2x 6 |> 5.

Solution:
Case 1: If 2x 6 0, then | 2x 6 |= 2x 6.
2x 6 > 5,

2x > 11,
11
x >
.
2

For this ase we want the values of x that satisfy both 2x 6 > 0
(our assumption) and x > 112 (our solution). This means x 3 and
11
x > 11
so
we
want
x
>
.
2
2

41

5.4 Absolute value

Case 2: If 2x 6 < 0 then 2x 6 = (2x 6).


(2x 6) > 5,
2x + 6 > 5,

2x > 1,
1
x < .
2

For this ase we want the values of x that satisfy both 2x 6 < 0
(our assumption) and x < 21 (our solution). This means x < 3 and
x < 21 so we want x < 12 . The solution to | 2x 6 |> 5 is x > 11
2 or
x < 21 .
Example 5.4.3.

Solve | x + 5 | 3.

Solution:
Case 1: If x + 5 0, then | x + 5 |= x + 5.
x + 5 3,

x 2.

For this ase we want the values of x that satisfy both x + 5 0 (our
assumption) and x 2 (our solution). This means x 5 and
x 2, so we want 5 x 2.
Case 2: If x + 5 < 0, then | x + 5 |= (x + 5).
(x + 5) 3,
x 5 3,
x 8,

x 8.

42

Chapter 5. Inequalities

For this ase we want the values of x that satisfy both x + 5 < 0 (our
assumption) and x 8 (our solution). This means x < 5 and
x > 8, so we want 8 x < 5.
The solution to | x + 5 | 3 is 5 x 2 or 8 x < 5,
whi h we an put together to obtain the solution 8 x 2.
5.5 Pra ti e Problems
Q 5.5.1.

1.

4x
7

Solve the following inequalities.

3+x

2. 12 6x < 8 x
3.

11a+4
6

> 3 + 2a

4.

7b14
b

Q 5.5.2.

Solve the following inequalities.

1. | 3x |> 12

2. | 4b 6 | 18
3. 7 | x + 14 |
4. 9 <| 3c 6 |

5.6 Answers to Pra ti e Problems


A 5.6.1.

(Q 5.5.1)

1. x 7

5.6 Answers to Pra ti e Problems

2. x >

4
5

3. a < 14

4. 0 < b 7
A 5.6.2.

(Q 5.5.2)

1. x > 4 or x < 4
2. 3 b 6

3. 21 x 7

4. c > 5 or c < 1

43

Chapter 6
Powers

6.1 Introdu tion

Consider the following equations


10 10 = 100,

10 10 10 = 1000,

10 10 10 10 = 10000.

Ea h of these equations an be written in a shorter form.


102 = 100,
103 = 1000,
104 = 10000.

In general, a produ t su h as
n terms

z
}|
{
a a a a

an always be expressed in the form an. In this ase a is known as the


base, and n the exponent, power, or index. This notation is simple to
use, and allows many al ulations to be performed more easily.
When performing operations using powers, there are several laws
that make al ulations easier. These are known as the index or power
45

46

Chapter 6. Powers

laws.
6.2 Index Laws
m

a a =a

am
mn
=
a
an
(a b)m = am bm

m+n

(am)n = amn
 a  m am
= m
b
b
1
an = n
a

a0 = 1

m
m/n
n
a
= ( a) = n am

Below we list ea h rule along with a brief illustration. In pra ti e, it


is su ient to simply apply the rule without in luding the intermediate
working.
1. Multipli ation

am an = am+n

is the statement:
m terms

n terms

m+n terms

z
}|
{ z
}|
{ z
}|
{
a a a a a a a a=a a a a

For instan e,

23 24 = (2 2 2) (2 2 2 2),
= 27 ,

= 23+4.

47

6.2 Index Laws

2. Division

am
mn
=
a
an

is the statement:
m terms

z
}|
{
a a a a
n terms

For instan e,

z
}|
{
a a a a

mn terms

z
}|
{
=a a a a

35
33333
=
,
32
33
= 33 ,
= 352.
3. Powers

am

n

= amn

is the statement:

m terms

z
}|
{

m terms

mn terms
}|
{ z
z
}|
{
a

a
n times
=a a a a

..

m terms

z
}|
{

a a a a

This illustration shows how the power law follows from repeated appli-

48

Chapter 6. Powers

ations of the multipli ation law.


(24)3 = 24 24 24,
= 24+4+4,
= 212,
= 243.
4. Produ ts

ab

For instan e,

m

= am bm

(3 5)2 = 3 5 3 5,
= 3 3 5 5,
= 32 52 .
5. Quotients

 a m
b

For instan e,

 3 3
5

am
= m
b

3 3 3
,
5 5 5
333
=
,
555
33
= 3.
5
=

6. Zero

a0 = 1

49

6.2 Index Laws

This illustration shows how the zero law follows dire tly from the quotient law.
31 3
0
11
3 =3
= 1 = = 1.
3
3
7. Negatives

an =

1
an

This illustration shows how the negatives law follows dire tly from the
quotient law and the zero law.
2

=2

03

20
1
= 3 = 3.
2
2

8. Fra tional Powers

m/n

m
n
= ( a) = n am

For instan e,
8

2/3

= (8

3
) = ( 8)2 = 22 = 4

1/3 2

or
8

2/3

2 1/3

= (8 )

It is important to remember that x

82

1/2

64 = 4

is equivalent to

x.

A ombination of these rules an be used to simplify many equations


involving powers. Remember that you an only ombine terms that
have the same base.
Example 6.2.1.

Solution:

Simplify

x4 x2
x3

x4 x2 x4+2 x6
= 3 = 3 = x63 = x3
3
x
x
x

50
Example 6.2.2.

Chapter 6. Powers

Simplify

Solution:

(x2 y 3 )2
x2 y 4

(x2 y 3 )2
x22 y (3)2
=
,
x2 y 4
x2 y 4
x4 y 6
= 2
,
x y4
= x4(2) y 64 ,
= x6 y 10 .

Example 6.2.3.

Solution:

Evaluate

29 23
16

29 23
29 23
=
,
16
24
293
=
,
24
26
= 4,
2
= 264,
= 22 ,
= 4.

Example 6.2.4.

Evaluate 272/3

51

6.3 Pra ti e Problems

Solution:
272/3 = (271/3)2,

3 1/3 2
= (3 )
,
2
= (331/3 ,

1 2
= (3 ,
= 32 ,
= 9.
Example 6.2.5.

Simplify

Solution:

(x3/2 y 2 )2
y 7/2x3

(x3/2 y 2 )2
x(3/2)2 y 22
=
,
y 7/2 x3
y 7/2 x3
x3 y 4
= 7/2
,
y
x3
= x33 y 4(7/2) ,
= x0 y (8/2)+(7/2),
= y 1/2,
1
= .
y

6.3 Pra ti e Problems


Q 6.3.1.

Write 95 as a produ t.

Q 6.3.2.

Simplify

1. x2 x3

2.

y5
y2

52

Chapter 6. Powers

3. (x4)3
4. (2x)3
5. x3
Q 6.3.3.

1.

Evaluate (without a al ulator)

213 24
214

2. 1252/3
3. (x y)0
Q 6.3.4.

Simplify

Q 6.3.5.

Simplify

Q 6.3.6.

Simplify

(x7 y 4 )2
(y 12 x3 )3

(x3 y)2
x4 y 6

214 93 51
2582 311

(provided x > 0, y 6= 0).

6.4 Answers to Pra ti e Problems


A 6.4.1.
A 6.4.2.

(Q 6.3.1) 9 9 9 9 9
(Q 6.3.2)

1. x5
2. y3
3. x12
4. 8x3
5.

1
x3

A 6.4.3.

(Q 6.3.3)

6.4 Answers to Pra ti e Problems

1. 8
2. 25
3. 1
A 6.4.4.
A 6.4.5.
A 6.4.6.

(Q 6.3.4) x23 y28


(Q 6.3.5) x y2

(Q 6.3.6) 28 35 53

53

Chapter 7
Logs

You should make sure you understand powers before reading; this se tion. For assistan e with powers refer to Chapter 6.
7.1 Introdu tion

Logarithms an be used to solve equations involving powers, as we shall


see in Se tion 7.4. We dene log10 y to be the power of 10 required to
obtain the result y . That is, log10 y = x is equivalent to the equation
10x = y . In this ase 10 is alled the base. For instan e, the equation
102 = 100 is equivalent to log10 100 = 2. So we have
log10 1000 = 3,
log10 100 = 2,
log10 10 = 1,
log10 1 = 0,
log10 0.1 = log10 (10)1,
= 1.

Determine log10 0.001.


1
Solution:
Let x = log10 0.001. Then 10x = 0.001 = 1000
=
103. Sin e we must have x = 3, we nd that log10 0.001 = 3.
Example 7.1.1.

55

56

Chapter 7. Logs

Determine log10 10k .


Solution:
Let x = log10 10k . Then 10x = 10k , and therefore
x = k . So log10 10k = k .
Example 7.1.2.

Logarithms are, by denition, the inverse of powers, so there are


many onne tions between the two on epts, some of whi h will be
des ribed later in this se tion.
No matter to what power 10 is raised, the answer is always greater
than 0. So log10 x is not dened for x 0.

We an also have logarithms to bases other than 10. For example,


log2 y is the power of 2 needed to obtain the result y , so log2 8 = 3.
In general if we have
y = ax ,

then this is equivalent to the equation


loga y = x.

In this ase a is the base, and the equation an be read as `the logarithm
of y to the base a is x'. It is required that a > 0, y > 0 and a 6= 1.

7.2 The number e

When working with logarithms, one important base is the number e,


whi h is signi ant in many elds. It appears in areas as diverse as ompound interest, radioa tive de ay and population growth. Although it

7.2 The number

57

may look like a variable, e is simply a number, with


e = 2.718281828459045235360287471352662497757
= 24709369995957496696762772407663035354759
= 45713821785251664274274663919320030599218
= 17413596629043572900334295260595630738132
= 32862794349076323382988075319525101901157
= 3834187930702154089149934884167509244 . . .

so
e 2.718281828.

Known as Euler's number1 , it is irrational so its de imal expansion


is innite and non-repeating. Like any other number is is easy to
perform simple al ulations with it, so e2 7.38906 and e3 20.0855.
The logarithm with base e is known as the natural logarithm, and is
sometimes written as ln rather than loge. Thus ln y = loge y is the
power of e needed to produ e y .
Example 7.2.1.

Solution:
ln e = 1.
Example 7.2.2.

Solution:

Determine ln e.
Let x = ln e. Then ex = e. Hen e x = 1, so
Determine ln

 
1
e

 
Let x = ln 1e . Then ex = 1e , so ex = e1 . Hen e

 
x = 1, so ln 1e = 1.
Example
1


3 2
7.2.3. Determine ln
e .

The number is named after Swiss mathemati ian Leonhard Euler

58

Solution:

so ln 3 e2 = 23 .

Chapter 7. Logs

3 2
Let x = ln e . Then ex = e2/3. Hen e x = 23 ,

Note that in these previous examples that the ln and e seem to an el


ea h other out. This is an example of a general rule, loga a = 1. In
parti ular this means that ln e = 1. The proof of this rule is obvious
from the denition of logarithms, as a1 = a.
A variety of bases an be used, but in pra ti e the most ommon are
e and 10. By onvention, if no base is given, the base is assumed to be
10. So log x refers to log10 x.
Note that both log10 and ln are easy to determine using your al ulator.
7.3 Logarithm Laws

Be ause of the onne tion between logarithms and powers, there are
analogues for many of the power laws. These logarithm laws below are
expressed using loga, as they are equally appli able for any base.
The logarithm laws are:

59

7.3 Logarithm Laws

loga x + loga y = loga x y

loga x loga y = loga

loga 1 = 0

x
loga a = x
logc b
loga b =
logc a

x

y

n(loga x) = loga xn
aloga x = x

The following examples demonstrate these logarithm laws. You an


use your al ulator to verify these al ulations.
Example 7.3.1.

Simplify ln 5 + ln 7.

Solution:

ln 5 + ln 7 = ln(5 7) = ln 35 3.5553.

Example 7.3.2.

Solution:

Simplify log10 24 log10 4. 


log10 24 log10 4 = log10

24
4

= log10 6 0.7782.

This next example demonstrates the reason that ln 1 = 0.

Evaluate ln x ln x in two ways: by subtra tion


and by the log laws.
Solution:
By subtra tion ln x ln x = 0. By the log laws
ln x ln x = ln xx = ln 1. We on lude that ln 1 = 0.

Example 7.3.3.

Example 7.3.4.

Simplify 2 log10 7 + log10 3.

60

Chapter 7. Logs

Solution:
2 log10 7 + log10 3 = log10 72 + log10 3,
= log10 49 + log10 3,
= log10(49 3),
= log10(147),

2.1673.
Example 7.3.5.

Solution:
Example 7.3.6.

Simplify

log10 81
log10 9
log10 81
log10 34
=
log10 9
log10 32

4 log10 3
2 log10 3

4
2

= 2.

Use the denition of logarithms to show that eln x =

x.

Solution:
Let y = ln x. By the denition, ey = x if and only
if y = ln x. Sin e y = ln x, this is ertainly true. It follows that
eln x = x.
Example 7.3.7.

Solution:
above.

Simplify eln(x1).
eln(x1) = x 1 by the nal logarithm law given

7.4 Solving Log Equations

Logarithms an be used to solve equations involving powers, by taking the logarithm of ea h side of the equation (with the same base).
Consider the example below.
Example 7.4.1.

Solve 10x = 4.

61

7.4 Solving Log Equations

Solution:
10x = 4,
log10 10x = log10 4,
x = log10 4,
0.6021 . . . ( from the al ulator ).

In general, we an onvert power equations into logarithm equation


and then apply the logarithm laws to redu e the equation into a form
for whi h a al ulator an provide an approximate solution.

Example 7.4.2.

Solve ex1 = 33.

Solution:
ex1 = 33,
ln(ex1 ) = ln 33,
x 1 = ln 33,

x = 1 + ln 33,
4.4965 . . . ( from the al ulator ).

It is also possible to solve equations whi h have a logarithm on one


side by applying the denition of logarithms.

Example 7.4.3.

Solution:

Solve log10(2x) = 1.4.

62

Chapter 7. Logs

log10(2x) = 1.4,
101.4 = 2x,
101.4
x =
,
2
12.559 . . . ( from the al ulator ).
Example 7.4.4.

Solution:

Solve log10 x + log10(x 3) = 1.

log10 x + log10(x 3) = 1,

log10 x(x 3) = 1,

x(x 3) = 101,

x2 3x 10 = 0,

(x 5)(x + 2) = 0,

x = 5 or x = 2.

Sin e the result in taking the logarithm of a negative number is not


dened, the ase x = 2 is dis arded. Therefore the solution is
x = 5.
It is a tually possible to solve power equations very easily using only
log10 or ln, both of whi h are fun tions on a standard al ulator. In
both of the examples below, either log10 or ln ould have been used for
the al ulations.
Example 7.4.5.

Solve 4x = 83.

63

7.4 Solving Log Equations

Solution:
4x = 83,
ln 4x = ln 83, ( taking ln on ea h side )
x ln 4 = ln 83,
ln 83
x =
,
ln 4
3.1875 . . . ( from the al ulator ).
Example 7.4.6.

Solve 11x = 27.

Solution:
11x = 27,
log10 11x = log10 27,
x log10 11 = log10 27,
log10 27
x =
,
log10 11
= 1.3744 . . . ( from the al ulator ).

For this type of question it is always a good idea to he k your


answer. For example, to he k the previous answer, evaluate 111.3744
with your al ulator and you should obtain 26.9954 . . . . This suggests
that, within rounding, the solution obtained was orre t.

Express log10 7 in base e.


Solution:
We use the hange of base law: loga b =
a = 10, b = 7, and c = e. Therefore
Example 7.4.7.

log10 7 =

ln 7
.
ln 10

logc b
logc a

. Here

64

Chapter 7. Logs

Cal ulate log2 18 using only log laws and the fa t


that log3 2 0.63093.
Solution:
Example 7.4.8.

log2 18 = log2(2 32),

= log2 2 + log2(32),

= 1 + 2 log2 3,
log 3
= 1+2 3 ,
log3 2
1
,
= 1+2
log3 2
2
1+
,
0.63093
4.16993.

7.5 Pra ti e Problems


Q 7.5.1.

Convert this logarithm equation into a power equation log7 55 =

x.
Q 7.5.2.

Convert these equations to equations involving logarithms.

1. 54 = 625
2. y = ex
3. 102 = 0.01
Q 7.5.3.

Simplify

1. log10 12 + log10 5
2. log10 44 log10 11

7.6 Answers to Pra ti e Problems

65

3. 2 ln 5 + ln 4 ln 10
4. log47 1
Q 7.5.4.

Simplify

2(log10 25)
log10 125

Solve the following (using a al ulator where ne essary, and


giving solutions to 4 de imal pla es).
Q 7.5.5.

1. 10x = 43
2. ex+1 = 12
3. ln x = 1.54
4. log10(3x 7) = 2.4
5. log10 x + log10(x 5) = 2
6. 3x = 19
7. 7x = 46
7.6 Answers to Pra ti e Problems
A 7.6.1.

(Q 7.5.1) 7x = 55.

A 7.6.2.

(Q 7.5.2)

1. log5 625 = 4
2. ln y = x
3. log10 0.01 = 2
A 7.6.3.

(Q 7.5.3)

66

Chapter 7. Logs

1. log10 60
2. log10 4
3. ln 10
A 7.6.4.

(Q 7.5.4) 43 .

A 7.6.5.

(Q 7.5.5)

1. 1.4849
2. 4.6646
3. 86.0629
4. 12.8078
5. 2.6801
6. 1.9675

Chapter 8
Trigonometry

8.1 Introdu tion

Trigonometry is on erned with the relationships between angles and


ratios of side lengths in triangles. You should be familiar with the three
trigonometri fun tions sine, osine and tangent (abbreviated to sin,
os and tan, respe tively). Ea h of these fun tions takes an angle as
input and returns a number. For example,

3
1
sin 60 =
,
cos 60 = , and
tan 45 = 1.
2
2
The numbers returned by these fun tions are ratios of side lengths of
a right-angled triangle.
8.2 Radians

You are probably used to measuring angles in degrees. Another ommon unit of measurement for angles is the radian. By denition, one
radian is the angle required to reate a se tor of a ir le with an are
length equal to the radius.

67

68

Chapter 8. Trigonometry

A ir le with radius r units has a ir umferen e of 2r units, so the


angle reated by a omplete revolution is equal to 2 radians. Thus
2 radians equals 360 degrees. From this We obtain the relationships
1 radian =

180
degrees ,

1 degree =

radians .
180

Thus 1 radian is approximately 57. It is helpful to memorise the


following angle measures: 90 = 2 radians, 60 = 3 radians, 45 = 4
radians, and 30 = 6 radians.

Convert 150 into radians.


Multiply 150 by 180 .

Example 8.2.1.

Solution:

150

Hen e 150 is equal to


ans).

5
6

150 5
=
= .
180
180
6

radians (that is, approximately 2.618 radi-

Convert 54 radians into degrees.

Multiply 54 by 180 .

Example 8.2.2.

Solution:

5 180 5 180

=
= 225.
4

Hen e

5
4

radians is 225.

8.3 Trigonometry of right-angled triangles

In a right angled triangle with an angle , we often refer to the sides


of the triangle in terms of their position relative to the angle .

69

8.3 Trigonometry of right-angled triangles

For onvenien e, we will refer to the lengths of the sides labeled opposite, adja ent and hypotenuse as opp, adj, and hyp, respe tively. The
three trigonometri fun tions sine, osine, and tangent are then dened
as
sin =

opp
,
hyp

cos =

adj
, tan
hyp

opp
.
adj

Note that for these denitions of the trigonometri fun tions, 0 < <

sin
.
Also
note
that
tan

=
.
2
cos

Determine sin , cos , and tan , where is the


angle marked in the following triangle.
Example 8.3.1.

Solution:
3
opp
6
, so sin =
= ,
hyp
10 5
adj
8
4
cos =
, so cos =
= ,
hyp
10 5
opp
6 3
tan =
, so tan = = .
adj
8 4
sin =

You may nd the a ronym SOH CAH TOA will help you in remembering the trigonometri ratios, where O stands for opposite, A stands

70

Chapter 8. Trigonometry

for adja ent, H stands for hypotenuse, and S, C, and T stand for sine,
osine, and tangent respe tively.
In many ases, given a parti ular angle, it is di ult to measure
side lengths of a triangle exa tly, so the best we an do is nd de imal
approximations for the trigonometri ratios. However, we do know
exa t values of the trigonometri ratios for ertain angles. Here are two
triangles that let you determine the exa t values pf the trigonometri
ratios for 30 = 6 radians, 45 = 4 radians, and 60 = 3 radians. To
help you in memorising these triangles, it may help to re all that in
any triangle, the shortest side is opposite the smallest angle.

Table 8.1:

Angles and Ratios in Important Triangles

Angle (deg) Angle (rad) sin cos tan

30
45
60

1
2
1
2
3
2

3
2
1
2
1
2

1
3

We an re-s ale these triangles, to have a hypotenuse of length l.


S aling a triangle does not hange the angles and does not hange the
ratios of the side lengths, although it does hange the side lengths.
These versions of the spe ial triangles will be useful later on.

8.3 Trigonometry of right-angled triangles

71

If you know an angle and one side length of a right-angled triangle,


you an use trigonometri ratios to determine the other side lengths.
Example 8.3.2.

Find the side length x in the following triangle.

Solution: We are given the length of the hypotenuse and x is the


length of the side opposite to the given angle, so the sine ratio is
used.
x
sin 30 = , so x = 12 sin 30.
12
From our spe ial triangles sin 30 = 12 , so x = 12 12 = 6. The length

of the side is 6 cm.


Example 8.3.3.

Find the side length y in the following triangle.

Solution:
We use tangent sin e the sides involved are opposite and adja ent to the angle marked 60.
tan 60 =

50
50
, so y tan 60 = 50, whi h means y =
.
y
tan 60

72

Chapter 8. Trigonometry

From our spe ial triangle, tan 60 = 3, so y =


length of the side is approximately 28.87 cm.

50

28.87. The

If you are given a triangle in whi h the given angle is not one of the
spe ial angles, then you will need to use your al ulator to determine
the relevant trigonometri ratio. When you use your al ulator, always
he k that you have it in the orre t mode (degrees or radians).
Example 8.3.4.

Find the side length z in the following triangle.

Solution:
We use osine sin e the sides involved are the hypotenuse and the side adja ent to the angle marked 23.
cos 23 =

z
, so z = 10 cos 23.
10

Now we use a al ulator to determine that z 9.2. The length of


the side is approximately 9.2 cm.
If you are given the value of a trigonometri ratio and asked to nd
the angle, you may be able to use the spe ial triangles. However if
the angle is not 30, 45 or 60 you will need to know how to use the
inverse trigonometri fun tions on your al ulator. These often appear
above the sin, os and tan buttons on your al ulator. They should
look like sin1, cos1 and tan1, or asin, a os and atan. You may have
heard these fun tions alled ar sin, ar os, and ar tan. The inverse

8.3 Trigonometry of right-angled triangles

73

trigonometri fun tions take a number as input and return an angle.


To use the inverse trigonometri fun tions to determine an angle in
a right-angled triangle you must know the lengths of at least two sides.
Example 8.3.5.

Determine the angle in the following triangle.

Solution:
The given side lengths are the adja ent and hypotenuse, so we use osine.
cos =

6
1
= .
12 2

We an refer to the spe ial triangles to nd that the angle with a
osine of 12 is 60. This is written mathemati ally as
= arccos

1
so = 60.
2

The angle is 60.


Example 8.3.6.

Determine the angle in the following triangle.

74

Chapter 8. Trigonometry

Solution:
The given side lengths are the opposite and the
hypotenuse so we use sine.
sin =

1
4
= .
16 4

The ratio 14 is not in the spe ial triangles, so we use a al ulator to


nd and angle whose sine is 14 .
= arcsin

1
so 14.
4

The angle is approximately 14.


8.4 General denitions of trigonometri fun tions

The unit ir le is the ir le that has a radius of 1 and is entered at


the origin of the Cartesian plane. We an use the unit ir le to dene
the trigonometri ratios for all angles (not just angles between 0 and
90).

Consider a point P on the ir umferen e of the unit ir le, so that the


angle measured anti lo kwise between the x-axis and the line segment
OP is . If P has oordinates (x, y), then the sine, osine, and tangent

75

8.4 General definitions of trigonometri fun tions

of the angle are dened as follows:


cos = x,

sin = y,

y
tan = .
x

sin
Noti e that tan = cos
, and that tan is not dened when x = 0,

sin e we annot divide by 0. Note that these denitions agree with the
previous denitions for 0 < < 2 .

Find sin 2 , cos 2 , and tan 2 .


Solution:
The point P that makes an angle of 2 radians measured anti lo kwise from the x-axis has oordinates (x, y) = (0, 1).
Example 8.4.1.

= 1.
2

cos = x so cos = 0.
2
y

tan =
so tan is not dened.
x
2
sin = y so sin

Find sin , cos , and tan .


Solution:
The point P that makes an angle of radians measured anti lo kwise from the x-axis has oordinates (x, y) = (1, 0).

Example 8.4.2.

sin = y so sin = 0.
cos = x so cos = 1.
y
0
tan =
so tan =
= 0.
x
1

From these denitions you an see the trigonometri fun tions have
dierent signs in dierent quadrants of the xy plane. Here are two
diagrams whi h may help you to remember in whi h quadrants the
trigonometri fun tions are positive and negative. The diagram on the

76

Chapter 8. Trigonometry

right is often alled the CAST rule (in ea h quadrant starting at the
lower right, a letter of the word CAST identies whi h of the fun tions
are positive in that quadrant; A means all are positive).

Below are the graphs for sine, osine and tangent. You an see from
the graphs that sine and osine have a period of 2 (so the graphs
repeat every 2 radians) and tangent has a period of radians. These
graphs are onsistent with the CAST diagram: you an see that sin
is positive between 0 and and negative between and 2 . From the
tangent graph you an see that the fun tion is not dened at 2 , whi h
we demonstrated in Example 8.4.1.

Suppose we want to nd the value of cos 34 . First we draw the point P
on the unit ir le whi h makes an angle of 3
4 measured anti lo kwise
from the x-axis. We must nd the x and y oordinates of P . To do this,
we draw a line from P to the x-axis to reate a right-angled triangle
with the right-angle on the x-axis. The angle that this right-angled
triangle makes at the origin is 4 .

8.4 General definitions of trigonometri fun tions

77

Using spe ial triangle OQP , we see cos 4 = 12 . Then taking into
a ount that the x- oordinate of a point in the se ond quadrant is

1
negative (hen e osine is negative) we see that cos 3
4 = cos 4 = 2 .

Evaluate sin 76 .
Solution:
We draw the point P to reate a right-angled triangle with an angle of 6 at the origin.
Example 8.4.3.

The y- oordinate of P is negative so sin 76 is negative. Using our


spe ial triangles we see that 76 = 12 .
Given a point P that reates an angle in the rst quadrant, there
are related angles in the other quadrants that orrespond to points
with x and y oordinates of the same magnitude, but possibly dierent

78

Chapter 8. Trigonometry

signs. Thus an angle and its related angles have the same trigonometri
values, ex ept possibly for a hange in sign.

Understanding related angles helps us nd all angles with a given


trigonometri ratio.
If you are asked to determine an angle that has a parti ular value of a
trigonometri ratio, there is not a unique answer. On the ir umferen e
of the unit ir le, there are two points with the same x- oordinate (two
angles with the same osine) and two points with the same y - oordinate
(two angles with the same sine). Also, if we allow angles greater than
2 radians, then we travel around the unit ir le more than on e and
the values of the ratios repeat themselves.
Suppose we were asked to nd all the values of between 0 and 2
radians su h that cos = 21 . First determine the angle in the rst
quadrant that has a osine of the same magnitude, that is, cos = 12 .
Using our spe ial triangles, this is = 3 . Now nd the related angles
in those quadrants where osine (x- oordinate) is negative. These are
=

2
4
=
and + = + = .
3
3
3
3

79

8.5 Pra ti e Problems

The two angles between 0 and 2 radians that satisfy cos = 12 are
4
= 2
and

=
.
3
3
Example 8.4.4.

sin = 23 .

Find the angles between 0 and 2 radians su h that

Solution:
The angle in the rst quadrant that has sin = 23
is 3 . The related angles in the quadrants where sine ( y- oordinate)
is positive are = 3 and = 3 = 23 .

The two angles that satisfy sin =

3
2

are =

and =

2
3

8.5 Pra ti e Problems

Here are Some problems for you to pra ti e on followed by answers.


Q 8.5.1.

Convert these angles to radians.

80

Chapter 8. Trigonometry

1. 120
2. 260
3. 315
Q 8.5.2.

1.

4
9

2.

3
5

3.

Q 8.5.3.

Convert these angles to degrees.

From this triangle, nd the requested values.

1. sin
2. cos
3. tan
Q 8.5.4.

Find the unknown variables.

1. Find the length of the side x in the following triangle.

8.5 Pra ti e Problems

81

2. Find the length of the side z in the following triangle.

3. Find the length of the angle in the following triangle.

Q 8.5.5.

Find the exa t values (where possible) for the following.

1. sin 11
6 .
2. cos 56 .
3. tan 43 .
4. sin 14
(use your al ulator for this one).
15
Q 8.5.6.

Find two angles in the range 0 to 2 su h that the following

are true.
1. sin =

2. cos = 1.
3. tan =

1
3

4. cos =

1
4

82

Chapter 8. Trigonometry

8.6 Answers to Pra ti e Problems


A 8.6.1.

1.

2
3

2.

13
9

3.

7
4

A 8.6.2.

(Q 8.5.1)

(Q 8.5.2)

1. 80
2. 108
3. 20
A 8.6.3.

(Q 8.5.3)

1. sin =

5
13

2. cos =

12
13

3. tan =

5
12

A 8.6.4.

(Q 8.5.4)

1. x = 10.62 cm
2. x = 30.91 cm
3. = 47.29
A 8.6.5.

1.
2.

1
2

3
2

(Q 8.5.5)

8.6 Answers to Pra ti e Problems

3. 3

4. 0.208
A 8.6.6.

1.

5
4

(Q 8.5.6)

and

7
4

2. 0 and 2
3.

and

7
6

4. 1.32 radians and 4.97 radians (approximately)

83

Chapter 9
Derivatives and Anti-derivatives

Derivatives and anti-derivatives appear in most areas of mathemati s,


and an understanding of them is vital. When maths is applied to
other elds, more often than not the pro ess of nding derivatives is
an integral omponent. Despite this, many derivatives are governed
by a small set of simple rules, whi h an make them quite simple to
al ulate.
This se tion ontains many referen es to fun tions. If you have not
ome a ross fun tions before just think of them like you would any
equations. You will learn more about fun tions within the ourse.
9.1 Basi derivatives

The derivative of a fun tion at a parti ular point is the instantaneous


rate of hange of the fun tion at that point. For a horizontal line, that
is a line f (x) = a, where a is a onstant, the rate of hange is always
zero. For a straight sloped line f (x) = ax, the rate of hange at all
points is just the gradient of the line, a. In fa t, there are 3 simple
rules that allow the derivative of any polynomial to be al ulated.
Rule 1.

f (x)axn gives f (x) = naxn1 if a is a onstant.


85

86

Chapter 9. Derivatives and Anti-derivatives

Rule 2.

f (x) = c gives f (x) = 0 if c is a onstant.

Rule 3.

f (x) = g(x) + h(x) gives f (x) = g (x) + h(x).

Let f (x) = 4. Find f (x).


Solution:
By Rule 2, f (x) = 0. (This makes sense be ause
f (x) = 4 is a horizontal line, so its slope is 0.)

Example 9.1.1.

Example 9.1.2.

Solution:
2 1x21 = 2x.

Let f (x) = x2. Find f (x).


Using Rule 1 with a = 1 and n = 2 gives f (x) =

Let f (x) = 3x2. Find f (x).


By Rule 1, f (x) = 2 3x21 = 6x.

Example 9.1.3.

Solution:

Let f (x) = 3x2 + 4. Find f (x).


Solution:
We know the derivatives of 3x2 and 4, so by Rule
3, f (x) = 6x + 0 = 6x.
Example 9.1.4.

Let f (x) = 3x2 + 5x 1. Find f (x).


Solution:
Let f (x) = g(x) + h(x) + k(x), with g(x) = 3x2,
h(x) = 5x and k(x) = 1. Then by Rule 3, f (x) = g (x) + h (x) +
k (x).
By Rule 1, as g(x) = 3x2, we know that g(x) = 2 3x21 = 6x.
Similarly, as h(x) = 5x, h(x) = 1 5x0 = 5.
Finally, by Rule 2, k(x) = 1 gives k(x) = 0.
So f (x) = g(x) + h(x) + k(x) = 6x + 5.
Example 9.1.5.

We an use the derivative of a fun tion to nd the gradient (or


slope) of the fun tion at any point. For example, onsider the fun tion

87

9.1 Basi derivatives

f (x) = 3x2 + 4. We know from Example 9.1.4 that the derivative of

this fun tion is f (x) = 6x. Say that we wanted to nd the gradient
at the point (4, 52). To do this, we substitute the value x = 4 into
the derivative, giving f (4) = 6 4 = 24. We say that the derivative
evaluated at x = 4 is equal to 24. This means that the gradient of the
fun tion f (x) = 3x2 + 4 at the point x = 4 is 24.
Example 9.1.6.

4x at x = 1.

What is the gradient of the fun tion f (x) = 2x2 +

Solution:
The gradient of the fun tion f (x) at any point is
the derivative f (x) at that point. From the rules above we have
f (x) = 4x + 4. So at x = 1, the gradient is f (1) = 4 + 4 = 0.
So at that point the gradient is zero.
Negative powers are handled in exa tly the same way as positive
powers.
Example 9.1.7.

Solution:

Let f (x) = x4 + 11x. Find f (x).


Let f (x) = g(x) + h(x), with g(x) = x4 and

h(x) = 11x.

We know that f (x) = g(x) + h(x).


Using Rule 1, g(x) = 4x41 = 4x5 and h(x) = 11.
So f (x) = g(x) + h(x) = 4x5 + 11.

Indeed, it is also possible to take derivatives this way when the powers involved are any rational numbers, not just integers.

Example 9.1.8. Let f (x) =


x. Find f (x).

Solution:
We know x = x1/2. So using Rule 1, f (x) =
1
1
1/2

x
=
.
2
2x1/2

88

Chapter 9. Derivatives and Anti-derivatives

If you have trouble with manipulating powers in the example above,


refer to Se tion 6.
You may noti e that any onstant in front of an xn term is essentially
un hanged when the derivative is taken. This follows from a general
rule, whi h governs the derivative of a term onsisting of a onstant
multiplied by a fun tion of x.
Rule 4.

a g (x).

Let f (x) = a g(x), with a a onstant. Then f (x) =

9.2 Derivatives of sin(x) and cos(x)

Often it is ne essary to take derivatives of the trigonometri fun tions.


The rules governing this are surprisingly simple. The two most signifi ant give the derivatives of sin(x) and cos(x).
Rule 5.

f (x) = sin x gives f (x) = cos x.

Rule 6.

f (x) = cos x gives f (x) = sin x.

Combining this with the previous rules allows a variety of derivatives


to be found.

Let f (x) = 3x2 + cos x + 4. Find f (x).


Using the rules above, we have f (x) = 6x sin x.

Example 9.2.1.

Solution:

Let f (x) = 4 sin x. Find f (x).


Sin e 4 is a onstant, f (x) = 4 cos x.

Example 9.2.2.

Solution:

9.3 Derivatives of

ex

and

ln x

89

9.3 Derivatives of ex and ln x

When learning about logarithms, the onstant e was introdu ed. The
rules for the derivatives of ex and ln x are also quite simple.
Rule 7.

f (x) = ex gives f (x) = ex .

Rule 8.

f (x) = ln x gives f (x) = x1 .

It is here that the signi an e of the number e be omes more apparent. The fun tion f (x) = ex has the unique property that f (x) =
f (x). So the gradient of the slope at any point along the fun tion is
equal to the value of the fun tion at that point.

Let f (x) = 3ex. Find f (x).


Sin e 3 is a onstant, f (x) = 3ex.

Example 9.3.1.

Solution:

One of the most signi ant appli ations of derivatives omes from
the fa t that they allow fun tions to be maximised or minimised. A
lo al maximum or minimum in a fun tion will orrespond to a position
where the gradient of the fun tion, and therefore the derivative of the
fun tion at that point, will be zero. Sin e fun tions an be used to
model all manners of problems in s ien e, business, e onomi s and information te hnology, this ability to nd extrema is extremely valuable.
Unlike derivatives, anti-derivatives are not unique. This is be ause the
derivative of a onstant is 0. So if F is the anti-derivative of f , then
F and F + c have the same derivative.

90

Chapter 9. Derivatives and Anti-derivatives

9.4 Leibniz's Notation

d
An alternative notation for f (x) is dx
f . In order to make the pro ess
d
and
of dierentiating f (x) easier, we use the Leibniz's notation dx
apply it to f (x) so that

f (x) =

We write

df
dx

d
f (x) .
dx

to abbreviate this.

Example 9.4.1.

Cal ulate

Solution:
f (x) =
=
=
=
=

d
dx f (x)

where f (x) = 4x3 + 7x 11.

d
f (x),
dx
d
(4x3 + 7x 11),
dx
d
d
d
(4x3) + (7x) + (11),
dx
dx
dx
d
d
d
4 (x3) + 7 (x) 11 (1),
dx
dx
dx
2
4(3x ) + 7 1 11 0,

= 12x2 + 7.

For the following, let a be a onstant, then we have the following

91

9.5 Anti-derivatives

rules of dierentiating using


da
=0
dx

d
d
af (x) = a f (x)
dx
dx
d sin x
= cos x
dx
dex
= ex
dx
Example 9.4.2.

Cal ulate

Solution:
f (x) =
=
=
=
=
=

d
dx :

dxn
= nxn1
dx
 df dg
d
f (x) + g(x) =
+
dx
dx dx
d cos x
= sin x
dx
d ln x 1
=
dx
x
d
dx f (x)

where f (x) = 12x5 + cos x ln x.

d
f (x),
dx
d
(12x5 + cos x ln x),
dx
d
d
d
(12x5) + (cos x) + ( ln x),
dx
dx
dx
d
d
12 (x5) sin x (ln x),
dx
dx
1
12(5x4) sin x ,
x
1
60x4 sin x .
x

9.5 Anti-derivatives

Anti-derivatives are the `opposite' of derivatives, and are also known as


indenite integrals. Where derivatives were used to nd the gradient
of fun tions, integrals an be used to nd the area under urves representing fun tions in the xy plane. The rules governing anti-derivatives

92

Chapter 9. Derivatives and Anti-derivatives

are very similar to those for derivatives in reverse. Anti-derivatives are


R
represented by the integral sign . On e again a few simple rules allow
the anti-derivatives of polynomials to be found.
R
n+1
n
f (x)dx = axn+1 + c, where c is an
Rule 1. f (x) = ax gives
arbitrary onstant.
Rule 2.

Rule 3.


R
R
R
f (x) + g(x) dx = f (x)dx + g(x)dx.
R

f (x)dx = f (x) + c, where c is an arbitrary onstant.

The third rule demonstrates an important point. Given the derivative of a fun tion, integration allows the original fun tion to be found.
Taking the derivative of a onstant gives a result of zero, however,
so there will be no referen e in the derivative to any onstant that
appeared in the original fun tion. For this reason an arbitrary onstant must be in luded for any integration, as in Rule l. This is why
anti-derivatives are known as `indenite' integrals.
R
f (x)dx.
Example 9.5.1. Let f (x) = 3x2 + 4x + 1. Find

Solution:
From Rule 2, we know the integral of this sum is
just the sum of the integrals of ea h individual term. Then using
Rule 1 ea h time we have
Z

3x2+1 4x1+1 1x0+1


f (x)dx =
+
+
+ c,
2+1 1+1 0+1
= x3 + 2x2 + x + c,

where c is an arbitrary onstant.

93

9.5 Anti-derivatives

Example 9.5.2.

Given f (x) = 9x4, nd three possibilities for

f (x).

Solution:

We know from Rule 3 that f (x) = f (x)dx. So


f (x) =

9x4dx,

9x4+1
=
+ c,
4 + 1
= 3x3 + c.

where c is an arbitrary onstant. So three possibilities for f (x) are


f (x) = 3x3 + 12,

f (x) = 3x3,

f (x) = 3x3 7.

This onne tion between derivatives and anti-derivatives allows us


to easily determine several other rules of integration. For instan e,
sin e we know that the derivative of sin x is cos(x), we know that the
integral of cos(x) is sin(x) (plus a onstant). This gives several new
rules for integration.
R
f (x)dx = cos (x) + c.
Rule 4. f (x) = sin (x) gives
Rule 5.

f (x) = cos (x) gives

Rule 6.

f (x) = ex gives

Rule 7.

f (x) =

1
x

gives

f (x)dx = sin (x) + c.

f (x)dx = ex + c.
f (x)dx = ln (x) + c.

In ea h of these rules c is an arbitrary onstant.

94

Chapter 9. Derivatives and Anti-derivatives

A ombination of these rules an be used to nd many integrals.


R

Let f (x) = 2x3 + x + ex + sin (x). Find f (x)dx.


Solution:
The integrals of the rst two terms an be determined by Rule l, and the integrals of the third and fourth terms an
be determined by Rules 6 and 4 respe tively. Combining this using
Rule 2, we have

Example 9.5.3.

Example

x4 x2
f (x)dx =
+
+ ex cos (x) + c.
2
2
R
1
9.5.4. Let f (x) = 2 + cos (x) + x . Find
f (x)dx.

Solution:
On e again the rules above allow the anti-derivatives
of ea h to be found. So
Z

f (x)dx = 2x + sin (x) + ln (x) + c.

9.6 Pra ti e Problems

Here are Some problems for you to pra ti e on followed by answers.


Q 9.6.1.

Find the derivatives of the following.

1. f (x) = 4x2 + 2x + 6
2. f (x) = 9x3 5x4 + 5

3. f (x) = x + 2x2
4. f (x) = a4 a2 + a
Q 9.6.2.

Find the derivatives of the following.

9.6 Pra ti e Problems

1. f (x) = 4 cos x
2. f (x) = 2 sin x + 8x
3. f (a) = a2 + sin a 4

4. f (x) = x3 3 cos x + 2x
Q 9.6.3.

Find the derivatives of the following.

1. f (x) = 5ex
2. f (x) = 7ex + x2
3. f (x) = 2 ln (x)

4. f (a) = 4a2 + ln (a)


Q 9.6.4.

Find the anti-derivatives of the following.

1. f (x) = 9x3 x + 7

2. f (x) = 3x2 + 2x + 8
3. f (x) = 4x3 + x3 + 3x
4. f (a) =
Q 9.6.5.

1
a2

+ 4a4 + 1

Find the anti-derivatives of the following.

1. f (x) = sin x + x2
2. f () = cos + 2
3. f (x) = 2x3 sin x

4. f (x) = 4x4 cos x


Q 9.6.6.

Find the anti-derivatives of the following.

95

96

Chapter 9. Derivatives and Anti-derivatives

1. f (x) = ex + 6x2
2. f (x) = 4x3 + x1
3. f (x) = cos x ex

4. f (x) = x1 sin x + 2x2


9.7 Answers to Pra ti e Problems
A 9.7.1.

(Q 9.6.1)

1. f (x) = 8x + 2
2. f (x) = 27x2 + 20x5
3. f (x) =

2 x

+ 4x

4. f (x) = 4a3 2a + 1.

See WS 12.9.1.
A 9.7.2.

(Q 9.6.2)

1. f (x) = 4 sin x

2. f (x) = 2 cos x + 8
3. f (a) = 2a + cos a
4. f (x) = 3x2 3 sin x + 2

See WS 12.9.2.
A 9.7.3.

(Q 9.6.3)

1. f (x) = 5ex

97

9.7 Answers to Pra ti e Problems

2. f (x) = 7ex + 2x
3. f (x) =

2
x

4. f (x) = 8a + a1
See WS 12.9.3.
A 9.7.4.

(Q 9.6.4)

1. f (x)dx =

9x4
4

x2 + 7x + c

2. f (x)dx = x3 + x2 + 8x + c
R

3. f (x)dx = 2x2 + x4 + 3x2 + c


4

4. f (a)dx = a1 + 4a5 + a + c
5

See WS 12.9.4.
A 9.7.5.

(Q 9.6.5)

1. f (x)dx = cos x + x3 + c
3

2. f ()d = sin + 2 + c
R

3. f (x)dx =
R

4. f (x)dx =

x4
2

+ cos x + c

4x3
3

See WS 12.9.5.
A 9.7.6.

sin x + c

(Q 9.6.6)

1. f (x)dx = ex + 2x3 + c
R

2. f (x)dx = x4 + ln x + c
R

3. f (x)dx = sin x ex + c

98

Chapter 9. Derivatives and Anti-derivatives

4. f (x)dx = ln x + cos x + 2x3 + c


See WS 12.9.6.

Chapter 10
Ve tors

Some quantities only require a single value, a magnitude, to spe ify


them. For example, 47 se onds, 10 m2 and 25 C ompletely spe ify
a time, area and temperature respe tively. These are known as s alar
quantities. There are other quantities, however, that require both a
magnitude and a dire tion to des ribe them. For instan e, velo ity
requires both a magnitude (like 10 km/h) and a dire tion (like NW).
The same is true for for e, magnitude and many other qualities. These
are known as ve tor quantities.
A ve tor is represented geometri ally in the (x, y) plane or in (x, y, z)spa e by a line segment of appropriate length ( alled its magnitude,
see Figure 10.1 below), printed in the orre t dire tion as indi ated
by an arrow. Only the length and dire tion of the representation are
signi ant: it an be pla ed anywhere onvenient in the (x, y) plane.

If P , Q are points in 2-spa e or 3-spa e, P Q denotes the ve tor form

P to Q. A ve tor v =P Q in the (x, y) plane may be represented by a


pair of numbers
!
!
v1
xQ xP
v=
=
,
v2
yQ yP

whi h is the same for all representations P Q of v. We all v1, v2 the


omponents of the ve tor v.
99

100

Chapter 10. Ve tors

Figure 10.1:

Ve tor of unit length at 45 to x-axis has many representations as shown

Figure 10.2:

Geometri representation of a ve tor

101

10.1 Position Ve tors

What is the ve tor v from the point P = (1, 3) to


the point Q = (4, 1) ?
!
!
!
Example 10.0.7.

Solution:

v=

We all the ve tor

0
0

xQ xP

y yP
! Q

41

1 3

the zero ve tor. It is denoted by 0.

10.1 Position Ve tors

Let P = (xP , yP ) be a point in the (x, y) plane. Then the ve tor OP ,


where O is the origin,1 is alled the position ve tor of P . Obviously
!

xP
OP =
.
yP
!
1
So the position ve tor of P = (1, 1) is just
.
1
10.2 Denition (magnitude)

For the ve tor v =P Q, the magnitude (or length or norm) of v, written


!
v1
k v k, is the distan e P Q between P and Q. Thus for v =
we
v2
have
q
k v k= P Q = v12 + v22.
!
3
Example 10.2.1. What is the magnitude of the ve tor v =
4
1

In a ve tor spa e, the origin O = (0, 0) is a spe ial point. This is not the ase in an ane spa e.

102

?
Solution:

Chapter 10. Ve tors

k v k=

10.3 Ve tor Addition

(3)2 + 42 = 9 + 16 = 25 = 5.

Geometri ally, we add ve tors by the triangle rule

Figure 10.3:

P Q + QR=P R

Consider the triangle P QR with v =P Q, w =QR, then v +

w =P R. In terms of omponents, if
!
!
v1
w1
v=
,
w=
,
v2
w2
then
v+w =

v1 + w1
v2 + w2

That is, algebrai ally we add ve tors omponent-wise.

103

10.4 S alar Multipli ation

Example 10.3.1.

Evaluate
7

Solution:

3
!
2
4

!
2
4

!
7 + (2)
(3) + 4

!
5
1

10.3.1 Properties

It follows from the omponent des ription that ve tor addition satises
the following2 properties:
v + w = w + v,
u + (v + w) = (u + v) + w,
v + 0 = 0 + v = v.

( ommutative law )

(10.3.1)

( asso iative law )

(10.3.2)

( exists zero ve tor )

(10.3.3)

10.4 S alar Multipli ation

With R a number (we will all it a s alar), we dene v to be the


ve tor of magnitude3
k v k=| | k v k

in the same dire tion as v if > 0 and opposite dire tion


if < 0.
!
v1
Using similar triangles it follows that if v =
, then
v2
!
v1
v =
.
v2
With numbers we might take these properties for granted. Of ourse 3 + 4 = 4 + 3 and 1 + (2 + 3) = (1 + 2) + 3 and
. However there are many important algebrai stru tures for whi h these properties may not hold.
3
The norm of a ve tor is always positive.
2

5+0=0+5=5

104

Chapter 10. Ve tors

Example 10.4.1.

Solution:

Evaluate 2v, where v =

2v =

(2) 4

(2) (2)

8
4

2
!

Note that that if we multiply any ve tor v =


obtain the zero ve tor:
0v =

0
0

v1
v2

by zero, we

= 0.

10.4.1 Properties

The three properties of ve tor addition (10.3.1), (10.3.2), and (10.3.3)


are not all of the properties ve tors must satisfy in order to be alled
ve tors. There are four properties involving s alar multipli ation and
addition. These are:
1v = v,
s1(s2v) = (s1s2)v,
s(u + v) = su + sv,

( exists 1 in s alars )

(10.4.1)

( s alar mult. is ommutative )

(10.4.2)

( s alar mult. distributes over addition )


(10.4.3)
(10.4.4)

(s1 + s2)v = s1v + s2 v.

10.5 Ve tor Subtra tion

We set
v = (1) v.

105

10.6 Unit Ve tors

That is, the ve tor with the same magnitude as v but pointing in the
opposite dire tion. Subtra tion of ve tors is then dened by
v w = v + (w).
!
!
4
2
Example 10.5.1. Evaluate

.
1
5
!
!
!
!
4
2
4
2
Solution:

=
+
=
1
5
1
5
!
6
6

4 2
1+5

10.6 Unit Ve tors

A unit ve tor is a ve tor of unit length (i.e. a ve tor that has magnitude
1). If v 6= 0 is a ve tor, then
v
=

v
kvk

determines a unit ve tor in the same dire tion as v.


Example 10.6.1.
!

ve tor v =

8
6

Determine a unit ve tor in the dire tion of the

. We rst ompute the norm of v and then divide v

by its norm so that the !new ve tor v will have norm 1.

Solution:

8
6

v
=

2
2
k= 8 + 6 = 100 = 10. So

v
1
=
k v k 10

8
6

4/5
3/5

106

Chapter 10. Ve tors

is a unit ve tor in the dire tion of

. While we are at it, we he k

6
p
p
2
2
that k v k= 1 : (4/5) + (3/5) = 16/25 + 9/25 = 1.

In parti ular

1
0

!
0
1

(10.6.1)

determine unit ve tors along the x and y axes respe tively.


v1 and v2 are alled the omponents
of v with respe t to i and j.
!
2
So the omponent form of
is 2i + 5j. The ve tors i and j are
5
alled oordinate ve tors.
10.7 Ve tors in 3-spa e

So far we!have dis ussed 2-spa e, the set of all 2- omponent ve tors
v1
v=
with real entries: v1, v2 R. Similarly in 3-spa e a ve tor
v2

v =P Q is represented in omponent form by


 v   xQ xP 
1
v = vv2 = yQyP ,
3

zQ zP

whi h is the same forall representations P Q of v.

v1
For v =OP = vv23 the magnitude of the ve tor v is
q

k v k= ON 2 + N P 2 = v12 + v22 + v32.

As before we add ve tors omponent-wise and we may dene multipli-

107

10.7 Ve tors in 3-spa e

ation by a s alar . So if
v 
1
v = vv2 ,
3

w=

w 
1

w2
w3

are ve tors, then we have the sum and s alar multiples respe tively
given by
 v1 +w1 
 v 
1
2 ,
v + w = v2 +w2 ,
v = v
R.
v3
v3 +w3

Example 10.7.1.

 6 
3
0

 1 

and w =

 12 

 11 

Evaluate v + 2w, where v =

Solution: v + 2w =

 1 
3
5

+2

Figure 10.4:

 6 
3
0

1
3
5

A ve tor in 3-spa e.

3
5

6
0

3
5

108

Chapter 10. Ve tors

The unit ve tors along the x, y , z axes are respe tively


 
 
 
1
0
0
i= 0 ,
j= 1 ,
k= 0 ,
0
0
1
v 
1
in terms of whi h a ve tor vv23 may be expressed as
v = v1i + v2j + v3k.

We all v1, v2, v3 omponents of v in the i, j, k dire tions respe tively.

10.7.1 Notation

We similarly have the zero ve tor


 
0
0= 0 .
0

2-spa e is usually denoted (when it is assumed that s alars are in


R) by R2 and 3-spa e by R3. For example
!
n x
o
2
R =
: x, y R .
y

A zeppelin is dire ted NE at 20 km/h into a


10 km/h wind in the dire tion E 60 S . Find the a tual speed4
and dire tion of the airship.
Solution: Using ve tors the a tual velo ity is v as shown in Figure
Example 10.7.2.

Speed is the magnitude of the velo ity ve tor.

109

10.7 Ve tors in 3-spa e

10.5, with v = z + w. Now


z = 20 cos 45i + 20 sin 45j,

= 10 2i + 10 2j.
w = 10 cos 60i 10 sin 60j,

= 5i 5 3j.

( using the relationship in right-angled triangles


cos =

adja ent side


,
hypotenuse

sin =

opposite side
).
hypotenuse

The omponent of v in the East (x) dire tion is

20 cos 45 + 10 cos 60 = 10 2 + 5.

The omponent of v in the North (y) dire tion is

20 cos 45 + 10 cos 60 = 10 2 5 3.

Therefore the magnitude of v is


q

2
kvk =
(10 2 + 5) + (10 2 5 3)2,
q

=
500 + 100 2(1 3),
19.91.

To al ulate the angle we use

10 2 5 3

tan =
.
10 2 + 5

110

Chapter 10. Ve tors

So 15.98.

Figure 10.5:

v gives the velo ity ve tor of the zeppelin.

10.8 Dot Produ t

For non-zero ve tors v =OP , w =OQ, the angle between v and w

is the angle with 0 180 between OP and OQ at the origin


point O.
10.8.1 Denition

The dot produ t or s alar produ t or inner produ t of ve tors v and


w, denoted by v w, is the number given by
(
0
if v or w = 0,
vw =
(10.8.1)
k v kk w k cos otherwise,
where is the angle between v and w.

111

10.8 Dot Produ t

The dot produ t may also be omputed as follows:


!
!
v1
w1

= v1w1 + v2w2,
v2
w2
v  w 
1
1
v2 w2
= v1w1 + v2w2 + v3w3 .
v3
w3

(10.8.2)
(10.8.3)

10.8.2 Properties

The dot produ t has the following properties.


1. If v and w 6= 0, then
v w = 0 if and only if k v kk w k cos = 0 if and only if = 90.

i.e. v w = 0 pre isely when v and w are perpendi ular (or


orthogonal) (write v w).
2. v v =k v kk v k cos 0 =k v k2, or

k v k= v v.
3.
v w = w v,

v (u + w) = v u + v w,

(10.8.4)
(10.8.5)

(v w) = (v) w = v (w), where R.(10.8.6)

112

Chapter 10. Ve tors

10.8.3 Component Form

If v = v1i + v2j + v3k and w = w1 i + w2j + w3 k, then we obtain


v w = (v1i + v2j + v3k) (w1i + w2j + w3k),
= v1i (w1i + w2 j + w3k)

this part is v1w1

= v3k (w1i + w2j + w3 k)

this part is v3w2

this part is v2w2

= v2j (w1i + w2j + w3 k)

= v1w1 + v2w2 + v3w3,

as we have
i i = j j = k k = 1 and

i j = j i = i k = k i = j k = k j = 0.

In parti ular, for v R3,


k v k2= v v = v12 + v22 + v32.
Example 10.8.1.

Find the angle between the following pairs of

ve tors:
(i) v =

Solution:
is given by

 
0
0
1

 
0
w= 2 .

(ii) v =

cos =

vw
.
k v kk w k

1
5
4


,

w=

 
3
3
3

From the denition of the dot produ t, the angle


(10.8.7)

10.8 Dot Produ t

113

(i) Here k v k= 1, k w k= 0 + 4 + 4 = 2 2,

   
0
0
v w = 0 2 = 0 + 0 + 2 = 2.
1

The formula (10.8.7) gives

1
2
cos = = .
2 2
2

So we have = arccos (1/ 2) = 45. (ii) v w = (1)(3) + (5)(3) +

(4)(3) = 3 15 + 12 = 0. Therefore the ve tors v and w are perpen-

di ular, hen e = 90.

If P = (2, 4, 1), Q = (1, 1, 1), R = (2, 2, 3),


nd the angle = P QR.

Solution:
We must nd the angle between the ve tors QP

and QR. Note that the ve tor QP is not given by the oordinates of

P or Q, but by the position ve tor OP OQ.


Example 10.8.2.

QP =

QP =

2
4
1

 2 
2
3

 
1
1
1

 
1
1
1

=
=

1
3
2

 3 
1
2

,
.

Therefore kQP k=kQRk= 1 + 9 + 4 = 14 , and the dot produ t


 1   3 

QP QR= 3 1 = 3 + 3 4 = 4.

It follows that

cos =

QP QR

kQP kkQRk

4 2
=
,
14
7

114

Chapter 10. Ve tors

and we obtain
106.60 = 106 + 0.60 = 106 36.

10.9 Pra ti e Problems


Q 10.9.1.

Whi h of the following quantities is a s alar quantity?

1. 36ms1 towards the entre of the Earth.


2. The sum of the rst seven prime numbers.
3. Eight kilometres per hour North-West.
Q 10.9.2.

Determine the magnitude of the ve tor u =

Q 10.9.3.

Given the ve tors u and v, where u =

12
!
3
2

and v =

1. Compare the norms of u and v.


2. Compare the dire tions of u and v.
Compute the !
magnitude of the ve tor w = u + v, where
1
1
u=
and v =
.

Q 10.9.4.
!

In Se tion 10.4,
! we state that using
! similar triangles it
v
v1
follows that if v = 1 , then v =
. Verify this with the
Q 10.9.5.

ve tors

!
2
3

and

6
9

v
!2

v2

115

10.9 Pra ti e Problems

Whi h of the following ve tors are unit ve tors?

Q 10.9.6.

1.

2
2

1/2

2.
3.
4.

1
1

3/2
!

0
1

Express the dire tion North-West as a unit ve tor given


that North orresponds to the dire tion of the ve tor (0, 1) and East
orresponds to the dire tion of the ve tor (1, 0).

Q 10.9.7.

Q 10.9.8.

Let u =

3
2

and v =

!
2
3

1. Show that u and v are orthogonal.


2. Express the ve tor

!
1

7
a s alar multiple of v.

as the sum of a s alar multiple of u and

3. Whi h 2- omponent ve tors with real entries may be expressed in


this way?
Q 10.9.9.

Let u
and v
be unit ve tors.

1. True or False: The dot produ t u


v
is also a unit ve tor?

2. Show that 1 u
v
1.

116

Chapter 10. Ve tors

3. What is the largest possible value that k u


+v
k an attain?

4. True or False: u
+v
and u
v
are orthogonal. Explain.
Q 10.9.10.

Let v =

2
4
2

1. Compute the norm of v.


2. Find a unit ve tor parallel to v.
3. Find a unit ve tor perpendi ular to v.
Let P = (1, 2, 3), Q = (2, 4, 7), R = (5, 1, 4). Cal ulate
the angle of the triangle P QR.
Q 10.9.11.

Q 10.9.12.

Consider the four points

P = (1, 6, 3), Q = (2, 9, 2), R = (6, 11, 0), S = (5, 8, 5).

What geometri al shape is the quadrilateral P QRS ?


Q 10.9.13.

Let v =

!
2
7

and w =

6
21

be ve tors in R2.

Whi h ve tors an be obtained from sums of s alar multiples of v


and w ?
10.10 Answers to Pra ti e Problems
A 10.10.1.

(Q 10.9.1) 2, 58 is a s alar quantity. See WS 12.10.1.

(Q 10.9.2) The magnitude of the ve tor is 13 units. See


WS 12.10.2.
A 10.10.2.

A 10.10.3.

12.10.3.

(Q 10.9.3) The ve tors u and v are parallel. See WS

117

10.10 Answers to Pra ti e Problems

A 10.10.4.

(Q 10.9.4) k w k= 4. See WS 12.10.4.

A 10.10.5.

(Q 10.9.5) See WS 12.10.5.

A 10.10.6.

(Q 10.9.6) 2 and 4. See WS 12.10.6.

A 10.10.7.

A 10.10.8.

(Q 10.9.7)

12 2

1
2 2

. See WS 12.10.7.

(Q 10.9.8) Part 1. u v = 0. Part 2.

1
7

19v). Part 3. All. Part 4. No. Part 5. If w is parallel to

1
13 (17u

!
1
2

, then

it is possible, otherwise not. Part 6. Convenien e. See WS 12.10.8.


(Q 10.9.9) Part 1. The dot produ t is a s alar. Part 2.
1 cos 1. Part 3. 2 units. Part 4. True. See WS 12.10.9.

A 10.10.9.




1
6
2 . Part 3.
A 10.10.10. (Q 10.9.10) Part 1. 2 6. Part 2. 6
1
 
0
w = 1 . See WS 12.10.10.
2

A 10.10.11.

WS 12.10.11.

(Q 10.9.11) 44.1286 or 0.77019 in radians. See

A 10.10.12.

(Q 10.9.12) A square. See WS 12.10.12.

A 10.10.13.

(Q 10.9.13) S alar multiples of v. See WS 12.10.13.

Chapter 11
Matri es

11.1 Introdu tion

Histori ally matri es were introdu ed to solve simultaneous linear equations of su h as:
2x 3y + z = 13,
x + 4y = 0,

x 2y + 3z = 2.

Su h systems of equations o ur in many real life problems. Sin e


then matri es have found many other appli ations in abstra t algebra,
quantum me hani s, applied mathemati s, hemistry, biology, omputer graphi s and engineering.
11.2 Denition (matrix)

A re tangular array of numbers

a11 a12

a21 a22
A=
...
...

am1 am2
119

. . . a1n

. . . a2n

. . . ... .

. . . amn

(11.2.1)

120

Chapter 11. Matri es

is alled an m n matrix - made up of m rows and n olumns. Entries


of the j -th olumn may be assembled into a ve tor

a1j

a2j

...

amj

alled the j -th olumn ve tor of A. Similarly


the entries of the i-th row

may be assembled into a row ve tor a1j a2j . . . amj alled the i-th
row ve tor.
aij is alled the (i, j) entry of A - ie. the number in the i-th row and
the j -th olumn. For brevity we write A = (aij ).
For example, the 2 3 matrix with entries (aij ) = (i j) is
!
0 1 2
A=
.
1 0 1
Note that an m 1 matrix is a olumn ve tor and a 1 n matrix
is a row ve tor.
The m n matrix A whose entries are all zero is alled the zero
matrix, denoted O; eg. the zero 3 2 matrix is

0 0

O=
0 0 .
0 0
Example 11.2.1.

Write a 3 2 matrix with A = (ai2j ).

121

11.3 Equality

Solution:

1 1

A=
4 8 .
9 18

11.3 Equality

Matri es A, B are said to equal, written A = B , if they are the same


size and aij = bij , for all i, j . To be the same size A and B must have
the same number of rows and the same number of olumns.
11.4 Addition

The sum of two m n matri es A and B is dened to be the m n


matrix A + B with entries
(a + b)ij = aij + bij

That is, we just add orresponding elements, (just like ve tor addi tion).
Addition is only dened between matri es of the same size.

Evaluate

Example 11.4.1.

1 4
6 1

6 3 + 0 3 .

9 4
2 0

122

Chapter 11. Matri es

Solution:

Sin e the addition is preformed element wise,




1 4
6 1
1 6 4 + 1
5 3




6 3 + 0 3 = 6 + 0 3 3 = 6 0 .


7 4
2 0
9 4
2 + 9 0 4

Evaluate

Example 11.4.2.

!
12 6
1

3 2 11

7 7 4

Solution:
This sum is undened, as it is not possible to add
matri es of dierent sizes.
11.5 Properties

For matri es A, B, C of the same size we have


A + B = B + A,
(A + B) + C = A + (B + C),
A+O =O+A=A
A + (A) = O

( ommutative law),
( asso iative law),
( exists zero matrix ),
( exists additive inverse ).

123

11.5 Properties

Using the matri es A, B and C below, prove the


asso iative law. That is, show that (A + B) + C = A + (B + C),
where
Example 11.5.1.

2 3

A = 1 4
,
6 1

Solution:

1 0

B = 4 3
,
8 7

3 1

C = 5 2

4 7

3 1
21 3+0

(A + B) + C = 1 + 4 4 + 3 + 5 2
,
6 + 8 1 + 7
4 7


3 1
1 3

= 5 7 + 5 2
,
4 7
2 8

1+3 3+1

=
5 + 5 7 + 2 ,
2+4 87

4 4

= 10 9
.
6 1

124

Chapter 11. Matri es

A + (B + C) =

3
1 + 3 0 + 1

+
4 4 + 5 3 + 2
,
1
8+4 77

3
2 1

4 + 9 5
,

6 1
12 0

2+2 3+1

1 + 9 4 + 5 ,

6 + 12 1 + 0

4 4

10 9 .

6 1

Therefore, you an see that (A + B) + C = A + (B + C).


11.6 S alar Multipli ation

Let A be an m n matrix and R . We dene A to be m n


with entries
(a)ij = aij , for all i, j.
Throughout we write A for (1) A. We may then dene subtra tion between matri es of the same size by
A B = A + (B)

125

11.7 Matrix Multipli ation

Let

Example 11.6.1.

A=

4 6 3
0 1 2

B=

5 1 0
3 1 0

Cal ulate A + B and A B .


Solution:
A+B =

1 5 3
3 2 2

AB =

If
A=

A+A=
1
A=
2

6 0
2 1

12 0

3 0 2

then
!

4 2
!
3 0
21

9 7 3

= 2A
etc.

11.7 Matrix Multipli ation

Let A = (aij ) be an m n and B = (bjk ) an n r matrix. Then AB


is the m r matrix with ik entry
(ab)ik = ai1b1k + ai2b2k + + ain bnk :

126

Chapter 11. Matri es

a11 a12 . . . a1n


b11 . . . b1k . . . b1r
.

...
... b . . . b . . . b
..

2k
2r

21

..
mr matrix
.
.
ai1 a22 . . . ain .
. = AB ,
.

...
... ...
...
...
..

am1 am2 . . . amn


bn1 . . . bnk . . . bnr
(11.7.1)

where (ab)ik = dot produ t between i-th row and A and k -th olumn
of B .
AB is only dened if the number of olumns A = the number of
rows of B .
learly, matrix multipli ation involves multiplying elements in rows

of the rst matrix by orresponding elements in olumns of the


se ond.

To multiply matrix A by matrix B (assuming it is possible):

1. Write matrix A.
2. Write matrix B

above

and to

the right

of matrix A.

3. Draw an empty answer matrix, to the right of A and below B .


This matrix will have the same number of rows as A, and the
same number of olumns as B .
4. Imagine a horizontal line through the rst row of A.
5. Imagine a verti al line through the rst olumn of B .
6. The entry in the answer matrix where the two lines interse t is
obtained by multiplying the rst row of A by the rst olumn

11.7 Matrix Multipli ation

127

of B ,
7. Repeat steps (4), (5), and (6) for ea h row of A and for ea h
olumn of B .

5 6
2 3 1 0

4 5

For example, let A = 0 4 3 2 and B =


3 4.

4 3 2 5
2 4
First he k the sizes and see that it is possible to nd the produ t
AB (A is 3 4 and B is 4 2, so the answer will be 3 2). Then
write the matri es as shown:

Note that the answer matrix is represented as a 3 2 re tangular array


of dots. Now, start drawing (or imagining) horizontal and verti al lines,
and look at where they interse t in the answer matrix.
The ell at position (1, 1) in the answer matrix is found by multiplying entries in the rst row of A by entries in the rst olumn of B
(and adding the produ ts to get the answer value). Continue until the
answer matrix is full.

128

Chapter 11. Matri es

11.8 Non- ommutativity of matri es

Even if AB and BA are both dened, in general AB 6= BA. That is


matrix multipli ation is non- ommutative.

Example 11.8.1.

Let A =

BA are dened.

0 1

2 1

and B =

Cal ulate AB and BA. Are they equal?


Solution:
AB =
=
=

0 1

1 2

2 1

3 4
!
0+3 0+4

23 44
!
3 4
.
1 0

1 2
3 4

. AB and

129

11.8 Non- ommutativity of matri es

Similarly
1 2

BA =

3 4

0 1

!
4 1

8 1

Note that AB 6= BA.

2 1

0 1

Example 11.8.2. Let A = 1 0 1 and B =


1 2 . A is
2 12
a 1 3 matrix and B is a 3 2 matrix so AB is dened but BA is


not dened sin e 2 6= 1. Cal ulate AB .


Solution:


 0 1

AB = 1 0 1
1 2 ,
2 12


1
= (0 + 0 2) 1 + 0 + 2 ,


= 2 32 .
Example 11.8.3.

Let

A= 1 3 9 ,

Cal ulate AB and BA.

B=
1 .
7

130

Chapter 11. Matri es

Solution:


 2

AB = 1 3 9
1 ,
7


= 2 + 3 + 63 ,
 
= 68 .

Not that if A were a olumn ve tor, this would be the dot produ t of
the two ve tors, but

2



BA =
1 1 3 9 ,
7

2 6 18

=
1 3 9 .
7 21 63

11.9 Properties of Matrix Multipli ation

For matri es of appropriate size,


1. (AB)C = A(BC),
2. (A + B)C = AC + BC ,
3. A(B + C) = AB + AC .
Another unusual property of matri es is that AB = 0 does not imply
that A = 0 or B = 0, ie. it is possible for the produ t of two non-zero

131

11.10 Transposition

matri es to be zero:
!
!
1 1
1 1
2 2

1 1

0 0
0 0

Also, if AC = BC , or CA = CB , then it is not true in general that


A = B.
If AC = BC , then AC BC = 0 and (A B)C = 0, but this does
not imply A B = 0 or C = 0.
11.10 Transposition

The transpose of an m n matrix A = aij is the n m matrix AT
with entries
aTji = aij ,
for all i, j

ie.

a11 a12

a21 a22

...
...

am1 am2

a11 a21

. . . a2n
= a12 a22
...
...
. . . ...

. . . amn
a1n a2n
. . . a1n

. . . am1

. . . am2

. . . ... .

. . . amn

So the row ve tors of A be ome olumn ve tors of AT and vi e versa.

132

Chapter 11. Matri es

For example, onsider the following matri es and their transposes.

!T
5 4

5 8 1
=
,
8 0

4 0 0
1 0
!T
!
1 1
1 2
=
,
2 0
1 0

7

T

=
7 5 2
5 .
2

11.10.1 Properties of Transposition

For matri es of appropriate size,


1. (A)T = AT , for R.
2. (A + B)T = AT + B T .
3. (AT )T = A.
4. (AB)T = B T AT , (not AT B T !).
11.11 Dot Produ t and Matrix Multipli ation

A olumn ve tor


v1

v2
n

v=
... R

vn

133

11.12 Square Matri es

may be interpreted as an n 1 matrix. Then the dot produ t (for two


olumn ve tors v and w) may be expressed using matrix multipli ation:

w1


 w
2
T

v w = v1w1 + v2w2 + . . . vn wn = v1 v2 . . . vn
... = v w.

wn

Also

v v = v12 + v22 + + vn2 =k v k2= vT v.




3
0


2
11

. Cal ulate the


Example 11.11.1. Let v =
and w =

7
4
4
2
dot produ t v w using matrix multipli ation.

Solution:


 11

v w = vT w = 3 2 7 4
4 = 42.

2

11.12 Square Matri es

An n n matrix

a11 a12 . . . a2n

a21 a22 . . . a2n

A=
...
... . . . ...

an1 an2 . . . ann

134

Chapter 11. Matri es

is alled a square matrix of order n. The diagonal ontaining the


entries
a11, a22, . . . , ann
is alled the prin ipal diagonal (or main diagonal) of A. If the entries
above this diagonal are all zero, then A is alled lower triangular. If
the entries below the prin ipal diagonal are all zero, then A is alled
upper triangular. For example,

a11 0 . . . 0 0
a11 a12
...
a2n

a21 a22 . . . 0 0 0 a22


...
a2n

..

...
... ... ...
...
...
.

...
... 0 0 0 . . . a
..

(n1)(n1) a(n1)n


an1 an2
. . . ann
0 0 ...
0
ann
lower triangular

upper triangular

If elements above and below the prin ipal diagonal are zero, so that
aij = 0,

then A is alled a diagonal

1 0

For example, A =
0 3
0 0

matrix
.

for i 6= j,

0
is a diagonal 3 3 matrix.
2

135

11.13 The Identity Matrix

11.13 The Identity Matrix

The n n identity matrix I = In, is the diagonal matrix whose entries


are all 1: ie.

1 0 0 ... 0

0 1 0 . . . 0

I = 0 0 1 . . . 0 .
. . .

.. .. .. . . . ...

0 0 0 ... 1

1. It is easily he ked that I a ts as the identity for n n matri es,


meaning:
I A = A I = A,
eg. for the 2 2 ase we have
!
!
1 0
a11 a12
I A =
=
a21 a22
0 1

a11 a12
a21 a22

2. More generally if A is m n, then


Im A = A In = A.

= A = A I.

136

Chapter 11. Matri es

3. The j -th olumn ve tor of I is the oordinate ve tor



0
.
..


0

ej =
1 j.

0
.
..

0

11.14 Inverses

The inverse of a square matrix A is the unique ( if it exists ) square


matrix B su h that AB = I = BA.
!
a b
There is a simple formula for the inverse of a 2 2 matrix A =
c d
given by
!
d b
1
B=
.
(11.14.1)
ad bc c a
This shows that we require that ad bc 6= 0 in order for the matrix B
to be dened. If ad bc 6= 0, then a 2 2 matrix A has an inverse,
and we say that A is invertible. The number D = ad bc is alled
the determinant.
You will learn about the inverse of a 33 matrix later in MATH1051.

137

11.15 Matrix Powers

11.15 Matrix Powers

For a square matrix A we may dene matrix powers


A2 = A A,

A3 = A2 A,

Ak+1 = Ak A,

and usually set A1 = A and A0 = I . The usual exponent laws hold:


eg.
Ak Ar = Ak+r ,
Example 11.15.1.

Solution:

Let A =

(Ak )r = Akr .
!
1 4
1

. Cal ulate A4.

A2 = A A, !
3 8
=
,
2 5
A4 = A2 A2, !
7 16
=
.
4
9

In identally, A1 is reserved to mean the inverse of A, whi h only exists if the determinant of A is non-zero. You will learn how to ompute
3 3 determinants later in MATH1051.

138

Chapter 11. Matri es

11.16 Pra ti e Problems


Q 11.16.1.

A=

Let

 4 1 4 
3 0 0
1 8 9

B=

5 5 2
6 1 4
2 2 11

Cal ulate the following if possible.

C=

 1 9

0
4 2 7
7 0 1

(11.16.1)

1. A + B
2. B + C
3. A + (B + C)
4. (B + A) + C
5. A C
6. B B
Let A, B , C be the matri es given in Eqn. (11.16.1).
Cal ulate the following if possible.
Q 11.16.2.

1. AO
2. A(B + C)
3. BC
4. 3C
5. 2B + 5C
6. A2

139

11.16 Pra ti e Problems

Q 11.16.3.

Let A, B , C be the matri es given in Eqn. (11.16.1),

and let
D=

!
3 0
1 0 0

12 0 9

, E=
,
I
=
. (11.16.2)
0 1 0
0 0

2 1 1
2 8
0 0 1

Cal ulate the following if possible.


1. AD
2. BE
3. B + E
4. D + E T
5. DC
6. C T DT
7. DI
8. IA
Q 11.16.4.

Let A =

1 7
2 1

. Find a matrix B su h that AB = I ,

where I is the 2 2 identity matrix. Is A the inverse of B ?


Q 11.16.5. Let A be the matrix given in Eqn. (11.16.1) and let
 1 
b = 8 . Write a system of three linear equations in the variables
0
x
x, y , z representing the matrix equation Ax = b, where x = yz .

140
Q 11.16.6.

Chapter 11. Matri es

Consider the system of linear equations


x + 2y = 13,
y 3z = 4,

(11.16.3)

5x 11y + 19z = 2.

Write a matrix equation whi h represents this system of equations.


Q 11.16.7.

Consider the system of linear s alar equations


2x + 3y = 4,

(11.16.4)

5x y = 2.

Use matri es and the denition of the inverse of a 2 2 matrix to


solve the system of equations (11.16.4).
11.17 Answers to Pra ti e Problems
A 11.17.1.

(Q 11.16.1)

1. A + B =
2. B + C =

9 4 6
3 1 4
3 10 20

6 4 2
2 3 11
9 2 10

3. A + (B + C) =
4. (B + A) + C =
5. A C =
6. B B =

 10 5

6
1 3 11
10 10 19

 10 5

6
1 3 11
10 10 19

3 8 4
1 2 7
6 8 10
000
000
000




141

11.17 Answers to Pra ti e Problems

See WS 12.11.1.
(Q 11.16.2)

A 11.17.2.

1. AO =

000
000
000

2. A(B + C) =
3. BC =
4. 3C =

6. A =

 62 11

37
18 12 6
71 38 180

 39 35 33 
26 56 3
87 14 3

3 27 0
12 6 21
21 0 3

5. 2B + 5C =
2

 17

 15 35

4
8 12 43
39 4 17

28 52
12 3 12
37 71 85

See WS 12.11.2.
A 11.17.3.

(Q 11.16.3)

1. AD annot be al ulated.

19 16

2. BE =
10 32.
28 88

3. B + E annot be al ulated.
4. D + E T =
5. DC =

15 0 11
2 1 7

!
75 108 9

9 20 6

142

Chapter 11. Matri es

75 9

T T

6. C D = 108 20
.
9 6

7. DI = D.
8. IA = A.

See WS 12.11.3.
A 11.17.4.

(Q 11.16.4) Let B =

1
13

verse of B . See WS 12.11.4.


A 11.17.5.

1 7

2 1

. Yes, A is the in-

(Q 11.16.5)
4x y + 4z = 1,
3x = 8,

x + 8y + 9z = 0.

See WS 12.11.5.
1

2 0
0 1 3
5 11 19

(Q 11.16.6) Let A =
Then we have AX = b. See WS 12.11.6.
A 11.17.6.

A 11.17.7.

,X=

x
y
z

,b=

16
(Q 11.16.7) x = 10
17 , y = 17 . See WS 12.11.7.

13
4
2

Chapter 12
Worked Solutions

12.1 Worked Solutions to Notation and Sets


WS 12.1.1. To Q 1.5.1. The empty set

know x 6 , and in parti ular this means

WS 12.1.2. To Q 1.5.2.

has
0 6 .

no elements at all. So for all numbers x we

1. The set {x | x Z and 2 x 6} ontains all the integers between and in luding 2 and
6. So this is 2, 3, 4, 5, and 6.
2. The set {x | x Z and 3 < x 0} ontains all the integers between and in luding 3
and 0, in luding 0 but not 3 So this is 2, 1 and 0.
3. The set {x | x N and x < 0} ontains all the natural numbers that are less than zero,
so this set is empty. So {x | x N and x < 0} = .

WS 12.1.3. To Q 1.5.3.

1. The set {3, 4, 5, 6, 7, 8} ontains all the integers between and in luding 3 and 8. So one
notation for this is {x | x Z and 3 x 8}.
2. The set {4, 2, 0, 2} ontains all the even integers between and in luding 4 and 2.
One notation for this is {x | x Z and x is even and x x 2}.
3. The set {6, 5, 3, 4, 5} ontains all the integers between 6 and 5 and all the integers
between 3 and 5. So one notation for this is {x | x Z and 6 x 5 or 3 x 5}.
4. The set {4, 5, 6, . . . } ontains all the natural numbers greater than, but not in luding 3.
One notation for this is {x | x N and 3 < x}.

WS 12.1.4. To Q 1.5.4.

1. The interval that onsists of all the real numbers between 3 and 5, in luding 3 and 5, is
simply denoted [3, 5], by denition.
2. The interval {x | x R and 7 x < 15} onsists of the real numbers between 7 and
15, where 7 is in luded but 15 is not. So this is notated [7, 15).
143

144

Chapter 12. Worked Solutions

3. The interval {x | x R and x > 4} onsists of the real numbers greater than but not
in luding 4. So this is notated (4, ).
4. The interval {x | x R and x 0} onsists of the real numbers less than or equal to 0.
So this is therefore notated (, 0].

12.2 Worked Solutions to Sigma Notation


WS 12.2.1. To Q 2.3.1.
1.
2.
3.

2n is the sum for n = 1, 2, 3 of 2n. This is 2 1 + 2 2 + 2 3, whi h is equal to


2 + 4 + 6 = 12.
P2
n
n
1
+ 30 + 31 + 32 , whi h is equal
n=1 3 is the sum for n = 1, 0, 1, 2 of 3 . This is 3
to 13 + 1 + 3 + 9 = 403 .
P5
i=0 i(i + 2) is the sum from i = 0 to 5 of i(i + 2). So this is simply
P3

n=1

0(0 + 2) + 1(1 + 2) + 2(2 + 2) + 3(3 + 2) + 4(4 + 2) + 5(5 + 2),

whi h is equal to 0 + 3 + 8 + 15 + 24 + 35.

WS 12.2.2. To Q 2.3.2.
1. The sum 3 + 4 + 5 + 6 + 7 + 8 is the sum of all integers from 3 to 8. In sigma notation
P
this is 8i=3 i.

2. The sum 2 + 1 + 4 + 7 is the sum of four terms, ea h of whi h represents an in rease


of 3 over the previous term, So if we onsider the rst term to be 0 3 2, (using 3
be ause this is the dieren e between onse utive terms) we an use sigma notation to get
P3
i=0 (3i 1).

1
3. The sum 1+ 12 + 13 + + 100
is the sum of 100 terms, ea h of whi h is a fra tion, where the
numerator is 1 and the denominator is the number between 1 and 100 whi h orresponds
P 1
to the parti ular term. In sigma notation this is 100
i=1 i .

WS 12.2.3. To Q 2.3.3.
j

j2

(1) (2j + n

).

This is

P6

j=3 (1)

(2j + nj2 )

orresponds to the sum with j = 3, 4, 5, 6 of

(1)3 (2 3 + n32 ) + (1)4 (2 4 + n42 ) + (1)5 (2 5 + n52 ) + (1)6 (2 6 + n62 ),

whi h is equivalent to (6 + n) + (8 + n2 ) (10 + n3 ) + (12 + n4 ).

WS 12.2.4. To Q 2.3.4. The sum 3x1 5x2 + 7x3 9x4 is a sum with alternating plus and
minus signs for ea h term, a fa tor out the front that in reases by 2 ea h time, and a subs ript
of x that in reases by 1 for ea h onse utive term. If we let the variable over whi h the sum is

12.3 Worked Solutions to Fra tions

145

to be performed be i, and allow this to go from 1 to 4, the rst term ould be 2i + 1xi . This
gives the right terms, but now they positives and negatives must be dealt with. Sin e (1)0 = 1
and(1)1 = 1, we an in lude the term (1)i1 to give the full sum. This gives a nal answer
P
of 4i=1 (1)i1 (2i + 1)xi .

12.3 Worked Solutions to Fra tions


WS 12.3.1. To Q 3.4.1.
1.
14x
2 7x
=
,
18x
2 9x
7
=
, sin e x 6= 0.
9

2.
9ab
9ab
=
,
27bc
27bc
9ab
=
,
9 3bc
a
=
, sin e b 6= 0.
3c

3.
8 + 4e
2(4 + 2e)
=
,
2e
2e
4 + 2e
.
=
e

WS 12.3.2. To Q 3.4.2.
1.
3
1
3
3
+
=
+ ,
12 4
12 12
3+3
=
,
12
6
=
,
12
1
=
.
2

146

Chapter 12. Worked Solutions

2.
4
1
4
1
+
=
+ ,
3 15
3 15
5
4
=
+ ,
15 15
5+4
=
,
15
9
=
,
15
33
=
,
35
3
=
.
5

3.
6 2
18 14

=
,
7 3
21 21
18 14
=
,
21
4
.
=
21

4.
13 7
13 14

=
,
18 9
18 18
13 14
=
,
18
1
=
.
18

WS 12.3.3. To Q 3.4.3.
1.
4
7
47

=
,
11 12
11 12
47
=
,
11 4 3
7
=
,
11 3
7
=
.
33

12.3 Worked Solutions to Fra tions

2.
8
3
9
3

=
,
13 9
13 8
39
=
,
13 8
27
=
.
104

3.
52
5 2

=
,
8 7
87
52
=
,
247
5
=
,
47
5
=
.
28

4.
3 5
3 6

=
,
8 6
8 5
36
,
=
85
323
=
,
245
33
,
=
45
9
=
.
20

WS 12.3.4. To Q 3.4.4.
1.
ab 2c
ab 2d 2c
+
=
+ ,
4
8d
4 2d
8d
2abd 2c
=
+ ,
8d
8d
2abd + 2c
=
,
8d
2(abd + c)
=
,
2 4d
abd + c
=
.
4d

147

148

Chapter 12. Worked Solutions

2.
4xy
3x
4xy 3x
+
=
+
,
9
12
33 34
3 3x
4xy 4
=
+
,
334 334
4xy 4 + 3 3x
=
,
334
16xy + 9x
=
.
36

3.
30gh 5h
30gh g 2 5h

,
14
7g
14 g
2 7g
2 3 5gh g 2 5h
=
,
2 7g
15g 2h 5h
=
.
7g

4.
9a
5
9a b
35

,
24 8b
24 b 3 8b
3 3a b 3 5
,
=
3 8b
3ab 5
=
.
8b

WS 12.3.5. To Q 3.4.5.
1.
12g h
12g h

=
,
16h 2
16h 2
3 4g h
=
,
16h 2
3g
=
.
8

2.
4e 9d
4e 9d

=
,
45
f
45 f
4e 9d
=
,
59f
4de
=
.
5f

149

12.4 Worked Solutions to Fa torisation

3.
x
xy 14
xy

=
,
7
14
7
x
xy 14
=
,
7x
xy 2 7
=
,
7x
= 2y.

4.
5a
7
5a 9a

=
,
7
9a
7
7
5a 9a
=
,
77
45a2
=
.
49

12.4 Worked Solutions to Fa torisation


WS 12.4.1. To Q 4.4.1.
1.
4y 2 16y = 4y y + 4 (4),
= 4y(y 4).

2.
32ab + 16b + 8abc = 8b 4a + 8b 2 + 8b ac,
= 8b(4a + 2 + ac).

3.
6xyz + 3yz + 18wyz = 3yz 2x + 3yz + 3yz 6w,
= 3yz(2x + 1 + 6w).

WS 12.4.2. To Q 4.4.2.

150

Chapter 12. Worked Solutions

1.
4 + 4b + b2 = 2 2 + 2 b + 2 b + b b,
= 2(2 + b) + (2 + b)b,
= (2 + b)(2 + b),
= (2 + b)2 .

2.
25a2 10ad + d2 = 25a2 10ad + d2 ,

= 5a 5a 5a d + 5a (d) + (d) (d),

= 5a(5a d) + (d)(5a d),


= (5a d)(5a d),
= (5a d)2 .

3.
x2 + 12xy + 36y 2 = x x + x 6y + x 6y + 6y 6y,
= x(x + 6y) + (x + 6y)6y,
= (x + 6y)(x + 6y),
= (x + 6y)2.

4.
144 48a + 4a2 = 12 12 + 12 (2a) + 12 (2a) + (2a) (2a),
= 12(12 + (2a)) + (12 + (2a))(2a),
= (12 + (2a))(12 + (2a)),
= (12 2a)2 .

WS 12.4.3. To Q 4.4.5.
1.
9y 2 36 = 3y 3y 6 6,

= (3y + 6)(3y 6).

12.4 Worked Solutions to Fa torisation

2.
64 121g 4 = 8 8 11g 2 11g 2 ,

= (8 + 11g 2)(8 11g 2).

3.
16x6 49y 8 = 4x3 4x3 7y 4 7y 4,

= (4x3 + 7y 4 )(4x3 7y 4).

WS 12.4.4. To Q 4.4.5.
1.
x2 5x + 4 = x x 1x 4x + (1) (4),
= x(x 1) + (x 1) (4),
= (x 1)(x 4).

2.
x2 + 2x 15 = x x + 5x 3x + (3) 5,
= x(x 3) + 5(x 3),
= (x 3)(x + 5).

3.
y 2 3y + 2 = y y 1y 2y + (1) (2),
= y(y 1) + (2)(y 1),

= (y 1)(y 2).

4.
x2 + 13x + 42 = x x + 6x + 7x + 6 7,
= x(x + 6) + 7(x + 6),
= (x + 7)(x + 6).

151

152

Chapter 12. Worked Solutions

5.
x2 11x + 24 = x2 3x 8x + (3) (8),

onsidering divisors of 24 : 1 24, 2 12, 3 8, 4 6


whi h sum to 11 : 3 8

= x(x 3) + (8)(x 3),

= (x 8)(x 3).

6.
a2 + 7a + 12 = a2 + 3a + 4a + 3 4,
= a(a + 3) + 4(a + 3),
= (a + 3)(a + 4).

WS 12.4.5. To Q 4.4.5.
1. Fa torisations ab of 3: are ab = 1 3. Fa torisations cd of 10 are 1 10, 2 5. We
require ac + bd = 17. This is 17 = 1 2 + 3 5. Therefore
3x2 + 17x + 10 = 1x 3x + 2 1x + 5 3x + 2 5,
= 1x(3x + 2) + 5(3x + 2),
= (x + 5)(3x + 2).

2. Fa torisations ab of 2: are ab = 1 2. Fa torisations cd of 16 are 1 16, 2 8, 4 4.


We require ac + bd = 12. This is 12 = 1 4 + 2 4. Therefore
2y 2 + 12y + 16 = 2y y + 4y + 8y + 4 4,

= 2y y + 4 2y + 4y + 4 4,

= 2y(y + 4) + 4(y + 4),


= (2y + 4)(y + 4).

3.
4x2 8x 12 = 4x2 8x 12,

= 4(x2 2x 3),

= 4(x2 + 1x 3x + (1) (3)),


= 4(x 3)(x + 1).

153

12.5 Worked Solutions to Inequalities

4.
2z 2 z 15 = 2z 2 z 15,

= 1 2z 2 3 2z + 5 1z 3 5,

= 2z(z 3) + 5(z 3),

= (z 3)(2z + 5).

12.5 Worked Solutions to Inequalities


WS 12.5.1. To Q 5.5.1.
1.
4x
3 + x,
7
4x 7 (3 + x),
4x 21 + 7x,

21 7x 4x,
21 3x,
7 x.

2.
12 6x < 8 x,

12 8 < 6x x,
4 < 5x,
4
< x.
5

3.
11a + 4
> 3 + 2a,
6
11a + 4 > 6 (3 + 2a),
11a + 4 > 18 + 12a,

4 18 > 12a 11a,


14 > a.

154

Chapter 12. Worked Solutions

4. First assume b > 0. Then


7b 14
5,
b
7b 14 5b,

sin e b > 0

7b 5b 14,
2b 14,
b 7.

Now assume b < 0. Then


7b 14
5,
b
7b 14 5b,

sin e b < 0

7b 5b 14,
2b 14,
b 7,

a ontradi tion sin e we assumed b < 0.

Sin e we annot have b = 0, therefore we must have 0 < b 7.

WS 12.5.2. To Q 5.5.2.

1. First assume that x > 0, then


| 3x | > 12,

3x > 12,
12
x >
,
3
x > 4.

Now assume that x < 0, then


| 3x | > 12,

3x > 12,
12
x <
,
3
x < 4.

Sin e x 6= 0 be ause this would mean that 0 > 12, whi h is absurd, we must have x > 4 or
x < 4.

155

12.5 Worked Solutions to Inequalities

2. First assume that 4b 6 > 0, (equivalently b > 32 ). Then


| 4b 6 | 18,
4b 6 18,

4b 18 + 6,
4b 24,
b 6,

whi h means that 32 < b 6. Now assume that 4b 6 0, (equivalently b 32 ). Then


| 4b 6 | 18,
4b + 6 18,

4b 18 6,
4b 12,

b 3.

Combining these inequalities, 3 b 6.

3. First assume x + 14 > 0 (equivalently x > 14). Then


7 | x + 14 |,
7 x + 14,

7 14 x,
7 x,

so that 7 x > 14. Now assume x + 14 0 (equivalently x 14). Then


7 | x + 14 |,
7 x 14,

7 + 14 x,
21 x,

21 x.

Combining these inequalities, 21 x 7.

156

Chapter 12. Worked Solutions

4. First assume 3c 6 > 0, (equivalently c > 2). Then


9 < | 3c 6 |,
9 < 3c 6,

9 + 6 < 3c,
15 < 3c,
5 < c.

Now assume 3c 6 0, (equivalently c 2). Then


9 < | 3c 6 |,
9 < 3c + 6,

9 6 < 3c,
3 < 3c,

1 > c.

Combining these inequalities, c > 5 or c < 1.

12.6 Worked Solutions to Powers


WS 12.6.1. To Q 6.3.1.

95

written as a produ t is 9 9 9 9 9.

WS 12.6.2. To Q 6.3.2.
1. x2 x3 = x2+3 = x5 .
2.

y5
y2

= y 52 = y 3.

3. (x4 )3 = x43 = x12 .


4. (2x)3 = 23 x3 = 8x3 .
5. x3 =

1
x3

WS 12.6.3. To Q 6.3.3.
1.

213 24
214

213+4
214

= 21714 = 23 = 8.

2. 1252/3 = (1251/3 )2 = ((53 )1/3 )2 = 53 3


1

3. (x y)0 = x0 y 0 = 1 1 = 1.

2

= 52 = 25.

157

12.7 Worked Solutions to Logs

WS 12.6.4. To Q 6.3.4.
(x7 y 4 )2
x72 y (4)2
=
x3(3) ,
(y 12 x3 )3
y 12(3)
x14 y 8
= 36
,
y
y 9
= x14(9) y 8(36) ,
= x23 y 28 .

WS 12.6.5. To Q 6.3.5.
s

(x3 y)2
=
x4 y 6
=
=
=

s
s
p
p

x(3)(2) y 2
,
x4 y 6
x6 y 2
,
x4 y 6
x64 y 2(6) ,
x2 y 4,

= xy 2 , (

provided x, y 0).

WS 12.6.6. To Q 6.3.6.
214 93 51
214 (32 )3 51
=
,
25 82 311
52 (23 )2 311
214 32(3) 51
=
,
52 232 311
214 36 51
=
,
26 311 52
= 2146 36+11 512 ,
= 28 35 53 .

12.7 Worked Solutions to Logs


WS 12.7.1. To Q 7.5.1.

log7 55 = x

is equivalent to 7x = 55 by the denition of logarithms.

WS 12.7.2. To Q 7.5.2.
1. 54 = 625 is equivalent to log5 625 = 4 by the denition.
2. y = ex is equivalent to ln y = x as ln y is the same as loge y .
3. 102 = 0.01 is equivalent to log10 0.01 = 2.

WS 12.7.3. To Q 7.5.3.

158

Chapter 12. Worked Solutions

1. log10 12 + log10 5 = log10 (12 5) = log10 60.


= log10 4.
2. log10 44 log10 11 = log10 44
11
= ln 10.
3. 2 ln 5 + ln 4 ln 10 = ln 25 + ln 4 ln 10 = ln 254
10

4. log47 1 = 0, as loga 1 = 0 for any a.

WS 12.7.4. To Q 7.5.4.

2 log10 25
log10 125

2 log10 52
log10 53

22 log10 5
3 log10 5

4
3

WS 12.7.5. To Q 7.5.5.
1.
10x = 43,
log10 10x = log10 43(taking

log of both sides)

x = log10 43(using log10 = 1)


1.6335 . . . (from

al ulator)

2.
ex+1 = 12,

(taking log of both sides)


x + 1 = ln 12 ( as ln e = 1)

ln ex+1 = ln 12

x = 1 + ln 12

1.4849 . . . (from

al ulator)

3.
ln x = 1.54,

(by the denitions of logarithms)


x 4.6646 . . . (from al ulator)

e1.54 = x

4.
log10 (3x 7) = 2.4,

102.4 = 3x 7

(by the denitions of logarithms)

2.4

3x = 10 + 7,
102.4 + 7
x =
,
3
86.0629 . . . (from

al ulator)

159

12.8 Worked Solutions to Trigonometry

5.
log10 x + log10 (x 5) = 2,

log10 x(x 5) = 102

(by the denitions of logarithms)

x2 5x 100 = 0,


5
5
x (1 + 17) x (1 17) = 0,
2
2
12.8078 . . . (the

log of a negative is not real)

6.
3x = 19,
log10 3x = log10 19

(taking log of both sides)

x log10 3 = log10 19,


log10 19
x =
,
log10 3
2.6801 . . . (from

al ulator)

7.
7x = 46,
ln 7x = ln 46

(taking log of both sides)

x ln 7 = ln 46,
ln 46
,
x =
ln 7
1.9675 . . . (from

al ulator)

12.8 Worked Solutions to Trigonometry


WS 12.8.1. To Q 8.5.1. To onvert degrees to radians, sin e 180 = , we multiply by 180
1. 120 180 =

260
360

2. 260 180 =

1320
920

3. 315 180 =

45 7
45 4

=
=
=

2
3

4
9

180

4180
9

= 80 .

2.

3
5

180

3180
5

= 108.

3.

180
9

= 20 .

= 1.

1.

180

180

WS 12.8.2. To Q 8.5.2.To onvert radians to degrees, sin e 180 = , we multiply by

= 1.

13
9
7
4

160

Chapter 12. Worked Solutions

WS 12.8.3. To Q 8.5.3.
1. sin =

OP P
HY P

5
13

2. cos =

ADJ
HY P

12
13

3. tan =

OP P
ADJ

5
12

WS 12.8.4. To Q 8.5.4.
1. cos 56 =

ADJ
HY P

x
19 cm

2. sin 15 =

OP P
HY P

8 cm
z

3. tan =

OP P
ADJ

13 cm
12 cm

. Therefore x = (cos 56 ) 19 cm = 10.62 cm.

. Therefore z =

8 cm
sin 15

= 30.91 cm.

. Therefore = arctan 13
= 47.29 .
12

WS 12.8.5. To Q 8.5.5. Ea h
of the following ex ept for (4) require onsideration of a right

triangle with sides equal to 1, 3, 2.


1. Sin e

the angle is in the fourth quadrant, where sine is negative. In fa t,


. This orresponds to a triangle with interior angle equal to 30 . We have
, so sin 11
= 1
.
6
2

3
< 11
4
6
11

= 6
6
sin 30 = 12

< 2 ,

2. Sin e 2 < 56 < , the angle is in the se ond quadrant, where osine is negative. This
orresponds to a triangle with interior angle equal to 30 . We have cos 30 = 23 , so
cos 5
= 2 3 .
6
3. Sin e < 43 < 34 , the angle is in the third quadrant, where tangent is positive. This
orresponds to a triangle with interior angle equal to 3 = 60 . We have tan 43 = tan 3 =

3.
4. Sin e 2 < 14
< , the angle is in the se ond quadrant, where sine is positive. Using the
15
al ulator, sin 14
= 0.207912 0.208.
15

WS 12.8.6. To Q 8.5.6.

1. The required triangle has sides 1, 1, 2 and the interior angles are 4 , 4 , 2 . Sin e sin < 0,
we have in the third or fourth quadrants. That is = + 4 = 54 and = 2 4 = 74 .
2. Sin e cos 0 = cos 2 = 1, = 0 and 2 .

3. The required triangle has sides 1, 3, 2 and the interior angles are 6 , 3 , 2 . The angle of
the triangle orresponding to tan = 13 is 6 . Sin e tan > 0, the angle is in the rst
or third quadrants so = 6 and = + 6 = 76 .
4. A al ulator or omputer gives arccos(1/4) 1.31812 however sin e cos > 0, is in
the rst or fourth quadrants. Therefore 1.32 radians and 2 1.31812 4.97
radians.

161

12.9 Worked Solutions to Derivatives and Anti-derivatives

12.9 Worked Solutions to Derivatives and Anti-derivatives


WS 12.9.1. To Q 9.6.1.
1. f (x) = 4x2 + 2x + 6 so f (x) = 4 2x21 + 2 1x11 + 0 = 8x + 2.
2. f (x) = 9x3 5x4 + 5 so f (x) = 9 3x31 5 (4)x41 = 27x2 + 20x5 .

3. f (x) = x + 2x2 so f (x) = 12 x1/21 + 2 2x21 = 21 x1/2 + 4x =

2 x

+ 4x.

4. f (x) = a4 a2 + a so f (x) = 4a41 2a21 + 1 = 4a3 2a + 1.

WS 12.9.2. To Q 9.6.2.

1. f (x) = 4 cos x so f (x) = 4 (1) sin x.


2. f (x) = 2 sin x + 8x so f (x) = 2 cos x + 8x11 = 2 cos x + 8.
3. f (a) = a2 + sin a 4 so f (a) = 2a21 + cos a = 2a + cos a.
4. f (x) = x3 3 cos x + 2x so f (x) = 3x31 3 (1) sin x + 2 = 3x2 3 sin x + 2.

WS 12.9.3. To Q 9.6.3.

1. f (x) = 5ex so f (x) = 5ex .


2. f (x) = 7ex + x2 so f (x) = 7ex + 2x21 = 7ex + 2x.
3. f (x) = 2 ln (x) so f (x) = 2 x1 = x2 .
4. f (a) = 4a2 + ln (a) so f (a) = 4 2a21 + a1 = 8a + a1 .

WS 12.9.4. To Q 9.6.4.
1.
2.
3.
4.

f (x)dx =

9
x3+1
3+1

1
x1+1
1+1

+ 71 x1 + c =

f (x)dx =

3
x2+1
2+1

2
x1+1
1+1

+ 81 x1 + c = x3 + x2 + 8x + c.

f (x)dx =

4
x3+1
3+1

1
x3+1
3+1

f (a)da =

1
a2+1
2+1

4
a4+1
4+1

+ a + c = a1 +

WS 12.9.5. To Q 9.6.5.

3
x1+1
1+1

9x4
4

x2
2

+ 7x + c.

+ c = 2x2 +
4a5
5

x4
4

3x2
2

+ c.

+ a + c.

1. f (x) = sin x + x2 so f (x)dx = cos x + x2+1 + c = cos x + x3 + c.


R

2+1

2
2. f () = cos + 2 so f ()d = sin + 1+1
1+1 + c = sin + 2 + c.

3. f (x) = 2x3 sin x so f (x)dx =


R

2
x3+1
3+1

4. f (x) = 4x4 cos x so f (x)dx =

WS 12.9.6. To Q 9.6.6.

cos x + c =

4
x4+1
4+1

x4
2

+ cos x + c.

sin x + c =

4x3
3

sin x + c.

162

Chapter 12. Worked Solutions

6
1. f (x) = ex + 6x2 so f (x)dx = ex + 2+1
x2+1 + c = ex + 2x3 + c.

2. f (x) = 4x3 + x1 so f (x)dx =

4
x3+1
3+1

+ ln x + c = x4 + ln x + c.

3. f (x) = cos x ex so f (x)dx = sin x ex + c.


R

2
4. f (x) = x1 sin x + 2x2 so f (x)dx = ln x cos x + 2+1
x2+1 + c == ln x + cos x + 2x3 + c.

12.10 Worked Solutions to Ve tors


WS 12.10.1. To Q 10.9.1. The sum of the rst seven prime numbers is 2+3+5+7+11+13+
17 = 58,

whi h is a s alar quantity sin e is only in luded magnitude, not dire tion. 36 m s1
towards the entre of the Earth is a ve tor quantity, provided it is understood from where the
starting point is. Eight kilometres per hour North-West is a ve tor quantity sin e it in ludes
magnitude and dire tion.

WS 12.10.2. To Q 10.9.2. The magnitude of the ve tor

u =

Pythagoras' theorem k u k= 52 + 122 = 169 = 13.

!
5
12

is determined using

WS 12.10.3. To Q 10.9.3.
kuk=
=

(3)2 + (2)2 ,

kuk=
kuk=

13,

(2)2 + (3)2 ,

13,

so these ve tors have the same norm. What about their dire tions? They have the same dire tions sin e there exists R su h that
3
2

!
3
=
.
2

This ve tor equation is equivalent to the simultaneous s alar equations


3 = 3,

2 = 2,

and we nd that = 1.

163

12.10 Worked Solutions to Ve tors

WS 12.10.4. To Q 10.9.4.
w = u + v,
!
!
1
1
=
+
,
2
2
!
11
=
,
2+2
!
0
=
.
4
kwk =

02 + 42 ,

= 4.

WS 12.10.5.! To Q 10.9.5. Consider


the two similar triangles
!

v =OQ=

2
3

, 3v =OQ =

6
9

, and

OP Q

and OP Q , where

O = (0, 0),

P = (2, 0),

Q = (2, 3),

O = (0, 0),

P = (6, 0),

Q = (6, 9).

The horizontal and verti al line segments of these triangles are the omponents of the ve tors
v and 3v.

WS 12.10.6. To Q 10.9.6. In order for the ve tor v to be a unit ve tor, we must have k v k= 1.
Letting

t=

!
2
,
2

u=

!
1/2

,
3/2

v=

!
1
,
1

w=

!
0
,
1

164

Chapter 12. Worked Solutions

we al ulate their norms:


ktk=
=

22

(2)2 ,

8,

= 2 2,

(1/2)2 + ( 3/2)2 , k v k = 12 + 12 , k w k = 02 + (1)2 ,


p

k u k = 1/4 + 3/4,
k v k = 2,
k w k = 1,

k u k = 1,
k v k = 2,
k w k = 1.
kuk=

It follows that only the ve tors u and w are unit ve tors.

WS 12.10.7. To Q 10.9.7. A ve tor orresponding to the dire tion North-West is v =


The orresponding unit ve tor is obtained by dividing by the norm of v,
1
1

1
v
=
v
=
kvk
2

WS 12.10.8. To Q 10.9.8. Part 1. To show that

uv =

!
21 2

.
1
2
2

u=

we ompute the dot produ t and show that this is zero.


3
2

1
1

3
2

and v =

2
3

are orthogonal,

!
2
,
3

= (3)(2) + (2)(3),
= 6 + 6,
= 0.

Therefore the ve tors


u and
!
! v are orthogonal.
!
1
3
2
Part 2. Let
=
+
. Then
7

3 2 = 1,
2 + 3 = 7.

Solving the system simultaneously gives =


1
7

Part 3. All 2- omponent ve tors

a
b

17
13

and = 19
. Therefore
13

1
(17u + 19v).
13

with a, b R may be expressed as

!
a
= u + v
b

165

12.10 Worked Solutions to Ve tors

be ause for any a, b R, we an always solve the system of simultaneous equations


3 2 = a,
2 + 3 = b.

WS 12.10.9. To Q 10.9.9. Part 1. The dot produ t u v is a s alar, not a ve tor so it annot

possibly be a unit ve tor.


Part 2. Show that 1 u v 1.

u
v
= k u kk v k cos ,
= cos

sin e u and v are unit ve tors ,

re all: 1 cos 1,
so 1 u v 1.
Part 3. The largest possible value that k u + v k an be o urs when u and v are equal, so that
ku
+v
k=k 2u k= 2.
!
!
u
v
To see that this is true in R2 , let u = 1 and v = 1 . Then sin e these are
u2
v2
2
2
2
2
k u k= 1 =k v k so that u1 + u2 = v1 + v2 .
!
u1 + v1
ku
+v
k = k
k,
u2 + v2
p
=
(u1 + v1 )2 + (u2 + v2 )2 ,
q
=
(u21 + u22 ) + 2(u1 v1 + u2 v2 ) + (v12 + v22 ),
p
=
1 + 2(u1 v1 + u2 v2 ) + 1,

=
2 1+u
v
,


0= 2 11
2 1 + 1 = 2 by Part 2 sin e 1 u
v
1.

To see that this is true in Rn , let u =

 u1 

..
.
u

and v =

 v1 

unit ve tors,

.. . Then sin e these are unit ve tors,


.
v
n

166

Chapter 12. Worked Solutions

k u k= 1 =k v k

so that u21 + u22 + . . . u2n = v12 + v22 + . . . vn2 .

 u1 +v1 
..
ku
+v
k = k
k,
.
un +vn
p
=
(u1 + v1 )2 + (u2 + v2 )2 + . . . (un + vn )2 ,
q
=
(u21 + u22 + . . . u2n ) + 2(u1 v1 + u2 v2 + . . . un vn ) + (v12 + v22 + . . . vn2 ),

2 1+u
v
,
=


0 = 2 11
2 1 + 1 = 2 by Part 2.

Part 4. We he k whether the ve tors u + v and u v are orthogonal by taking their dot produ t.
Re all that if it is zero, then they are orthogonal.
(
u+v
) (
uv
) = u
u
u
v
+v
u
v
v
,
= u
u
u
v
+u
v
v
v
,
= u
u
v
v
,

= k u k2 k v k2 ,
= 1 1,
= 0.

Therefore u + v and u v are orthogonal, True.

WS 12.10.10. To Q 10.9.10. Part 1. With v =


k v k=

Part 2.

2
4
2

22 + 42 + (2)2 = 24 = 2 6.

 
v
6 1
1  2 
4
2
v
=
= 2
=
,
kvk
6 1
2 6

whi h is a unit
v.
 ve tor parallel to v. Another possible answer is
w1
3
Let w = ww23 R be perpendi ular to v. Then
vw =

2
4
2

  w1 
2
w
,
w3

= 2w1 + 4w2 2w3 ,


= 0.

If we assume that w1 = 0, then 4w2 = 2w3 so that if we let w2 = 1, then


 we must have
0
w3 = 2. Therefore, among innitely many su h ve tors, the ve tor w = 1 is perpendi ular
2

to v =

2
4
2

167

12.10 Worked Solutions to Ve tors

WS 12.10.11. To Q 10.9.11. Let

u =RP ,

v =RQ,

= P R,
   5 
1
= 2 1 ,
4
 34 
= 3 ,

= Q R,
   5 
2
= 4 1 ,
4
 73 
= 5 .
3

Let be the angle between u and v. Then

u v = (4)(3) + (3)(5) + (1)(3),


= 12 + 15 3,

= 24.

u v = k u kk v k cos ,
p
p
24 =
(4)2 + 32 + (1)2 (3)2 + 52 + 32 cos ,

=
26 43 cos ,
24
cos = ,
26 43
0.717778,
44.1286

or 0.77019 in radians.

WS 12.10.12. To Q 10.9.12. Consider the four points


P = (1, 6, 3),

Q = (2, 9, 2),

R = (6, 11, 0),

S = (5, 8, 5).

If we were to onne t the points P to Q with a line segment, Q to R, R to S , and S to P , what


geometri gure would we have produ ed?

P Q = Q P,
   1 
2
= 9 6 ,
3
 221 
= 96 ,
 2+3

1
3
=
,
5

QR = R Q,
   
6
2
= 11 9 ,
0
 62  2
= 119 ,
 02

4
2
=
,
2

RS = S R,
 5   
6
= 8 11 ,
5
 56  0
= 811 ,
 50

1
3
=
,
5

= P Q,

SP = P S,
 1   5 
= 6 8 ,
3
5
 15 
= 68 ,
 3+5

4
2
=
,
2

= QR .

We have produ ed a square sin e we have anti-parallel ve tors of equal length.

168

Chapter 12. Worked Solutions

WS 12.10.13. To Q 10.9.13. Sin e the ve tors

2
7

v =

and w =

w = 3v, we an only obtain s alar multiples of v from sums of


Let , R, then the sums of s alar multiples of v and w are

6
21

s alar multiples of v and w:

v + w = v 3v,

= ( 3)v,

= v,

where = 3 R,

whi h is a s alar multiple of v.

12.11 Worked Solutions to Matri es


Re all that
A=

 4 1 4 
3 0 0
1 8 9

B=

WS 12.11.1. To Q 11.16.1.
1. A + B =

 4 1 4 
3 0 0
1 8 9

5 5 2
6 1 4
2 2 11

5 5 2
6 1 4
2 2 11

 4+5 1+5

4+2
36 0+1 0+4
1+2 8+2 9+11

C=

are parallel,

9 4 6
3 1 4
3 10 20

 1 9

0
4 2 7
7 0 1


.

169

12.11 Worked Solutions to Matri es

2. B + C =

5 5 2
6 1 4
2 2 11

3. A + (B + C) =

 1 9

0
4 2 7
7 0 1

 4 1 4 

3 0 0
1 8 9

 4 1 4 

3 0 0
1 8 9

 1

9 0
4 2 7
7 0 1

6. B B = bij bij = 0 =

WS 12.11.2. To Q 11.16.2.
1. AO = aij 0ij =


2. A(B + C) =
3. BC =

4. 3C = 3

 1 9

3 0 0
1 8 9

 1 9

5 5 2
6 1 4
2 2 11

3 0 0
1 8 9

0 0 0
0 0 0
0 0 0

4+2
32 0+3 0+11
1+9 8+2 9+10

6
1 3 11
10 10 19

 41 1+9 4+0 
34 02 07
17 8+0 9+1

45+10+0 0+352
6+4+28 54+2+0
0+74
2+8+77 18+4+0 0+1411
30 
37
3(1)

 1 9

0
4 2 7
7 0 1

=


3 27 0
12 6 21
21 0 3

2. BE =

12+0+0 3+0+0 12+0+0


4+24+9 1+0+72 4+0+81

12 0
9
,
2 1 1

5 5 2
6 1 4
2 2 11

 10 5

6
1 3 11
10 10 19

3 8 4
1 2 7
6 8 10

3 0

E = 0 0 ,
2 8

 62 11

37
18 12 6
71 38 180

 39 35 33 

 15 35

26 56 3
87 14 3

10+5 1045 4+0


12+20 2+10 8+35
4+35
4+0 225

 163+4 4+0+32 16+0+36 

6 4 2
2 3 11
9 2 10

163+8 811+40
18+0+0 12+0+0 6+0+0
616+81 4+24+18 2+88+90

1. AD annot be al ulated sin e 3 6= 2.




 24+2+36

WS 12.11.3. To Q 11.16.3. Re all


D=

 4+6 14

 5+20+14

32
30

+5

34
37

5+1 59 2+0
6+4 1+2 4+7
2+7 2+0 111

 10 5

 31 3(9)

 4 1 4  4 1 4 
3 0 0
1 8 9

6 4 2
2 3 11
9 2 10

0
4 2 7
7 0 1

0
4 2 7
7 0 1

0 0 0
0 0 0
0 0 0

 4 1 4 

5 5 2
6 1 4
2 2 11

5. 2B + 5C = 2
6. A2 =

6 4 2
2 3 11
9 2 10

4. (B + A) + C = A + (B + C) =
5. A C =

 17

4
8 12 43
39 4 17

28 52
12 3 12
37 71 85

1 0 0

I = 0 1 0 .
0 0 1

3 0
15 + 0 + 4 0 + 0 + 16
19 16

0 0 = 18 + 0 + 8 0 + 0 + 32 = 10 32.
2 8
6 + 0 + 22 0 + 0 + 88
28 88

3. B + E annot be al ulated sin e B is a 3 3 matrix and E is a 3 2 matrix.


4.

12 0
9
D + ET =
2 1 1
!
15 0 11
=
.
2 1 7

3 0 2
0 0 8

12 + 3 0 + 0
9+2
2 + 0 1 + 0 1 + 8

170

5.

6.

Chapter 12. Worked Solutions

!
12 0
9  1 9 0 
4 2 7
DC =
=
7 0 1
2 1 1
!
75 108 9
=
.
9 20 6

75
9

C T D T = (DC)T = 108 20.


9
6

!
12 + 0 + 63 108 + 0 + 0 0 + 0 9
,
247
18 2 + 0 0 7 + 1

7. DI = D.
8. IA = A.

WS 12.11.4. To Q 11.16.4. Re all that

1 7
2 1

A=

the inverse of a 2 2 matrix, Eqn. (11.14.1). B =

a b
c d

. We apply the formula for

d b
c a

1
adbc

is the inverse of B sin e AB = I = BA.

1 7
2 1

1
13

. Yes, A

WS 12.11.5. To Q 11.16.5.
Ax =

Therefore

 4 1 4  x 
3 0 0
1 8 9

y
z

 4xy+4z 
3x
x+8y+9z

 1 
8
0

4x y + 4z = 1,
3x = 8,

WS 12.11.6. To Q 11.16.6. Let


AX = b.

x + 8y + 9z = 0.
1 2 0 
x
 
13
A = 0 1 3 , X = yz , b = 4 .
5 11 19

Then we have

WS 12.11.7. To Q 11.16.7. The methods learned in Q ?? have shown that in order to solve
2x + 3y = 4,
5x y = 2,

, we must solve the matrix equation


2 3
5 1

x
y

for whi h we require the inverse A1 of the matrix A =

!
4
,
2
2 3
5 1

. The formula for the inverse

171

12.11 Worked Solutions to Matri es

of a 2 2 matrix, Eqn. (11.14.1), shows that A1 =


1
17

1 3
5 2

2 3
5 1

1
17

x
y

1
=
17

x
y

1
=
17

x
y

1
17

1 3
5 2

1 3
5 2

!
4
,
2

!
4+6
,
20 4
!
10
.
16

Therefore the solution to the system of equations (11.16.4) is x = 10


, y = 16
.
17
17

Referen es
[1 P. Abbott, Trigonometry, St. Paul's House, London, 1970. UQ Library: SSAH, QA533 .A2 1970
[2 F. P. Beer, E. R. Johnston, Jr., Ve tor me hani s for Engineers, M Graw-Hill Ryerson, UQ Library: Engin
& S ien es, TA350 .B3553 1990-V.2
[3 A. A. Blank, Problems in al ulus and analysis, Wiley, New York, 1966. UQ Library: Engin & S ien es,
QA303 .B66 1966
[4 J. B. Fraleigh, A rst ourse in abstra t algebra, 7th Ed., Pearson Edu ation, 2003. UQ Library: Engin &
S ien es, QA162.F7 2003
[5 A. G. Hamilton, Linear Algebra  An Introdu tion with on urrent examples, Cambridge University Press,
1989. UQ Library: Engin & S ien es, QA184.H362 1989
[6 A. I. Kostrikin, Y. I. Manin, Linear Algebra and Geometry, Gordon and Brea h S ien e Publishers, 1989.
UQ Library: Engin & S ien es, QA184.K69 1989
[7 E. Kreyszig, Advan ed engineering mathemati s, 8th Ed., John Wiley & Sons, 1999. UQ Library: Engin
& S ien es, QA401 .K7 1999
[8 R. Larson, B. H. Edwards, D. C. Falvo, Elementary Linear Algebra, 5th Ed., 2004. UQ Library: Engin &
S ien es, QA401 .L39 2004
[9 C. W. D. Rad lie, K. A. Dan, Geometry and Cal ulus III, Brooks Waterloo, Milton, Qld., 1990. UQ
Library: Engin & S ien es, QA459 .R43 1990
[10 C. W. D. Rad lie, K. A. Dan, Matri es and ve tors, William Brooks, Brisbane, 1975. UQ Library: Engin
& S ien es, QA184 .R34 1975
[11 MATH1050 Mathemati al Foundations Course Notes, Semester 2, 2014. Available for sale at pod:
http://www.pod.uq.edu.au/

[12 MATH1061

Dis rete Mathemati s,


http://www.pod.uq.edu.au/

Semester

2,

2014.

Available

for

sale

at

pod:

[13 J. Stewart, Cal ulus, Brooks/Cole-Thomson Learning, Belmont, CA, 2003. UQ Library: Engin & S ien es,
QA303 .S8825 2003
[14 G. Strang, Introdu tion to linear algebra, Wellesley-Cambridge Press. 1998. UQ Library: Engin & S ien es,
QA184 .S78 1998 https://library.uq.edu.au/

173

S-ar putea să vă placă și